E Maths Sec 42011
E Maths Sec 42011
Secondary School
Examination
Papers
5 A man bought a watch for $560. He made a profit of 15% of the cost price after selling the
watch at a discount of20% of the selling price. Find the actual selling price of the watch.
Answer
~------------
[2]
[Turn Over
Anglican High School Preliminary Examinations 03 2011 Math (4016/01)
4
3
{b) The actual floor area of the school hall is 700 m 2, find the corresponding floor area of
the hall of the model in cm2 .
[Turn Over
Anglican High School Preliminary Examinations 03 2011 Math (4016/01)
5
4
Answer
-5 -4 -3 -2 -1 0 1 2 3 4 5
[2]
9 A gardener decides to water the plants every three days, trim them every four days and
fertilise them every eight days. If he perfonns these 3 tasks on Day 1, list the days that the
gardener will carry out at least 2 tasks over a 30-day period.
[2)
(Turn Over
Anglican High School Preliminary Examinations 03 2011 Math (4016/01)
6
5
B ,,,
10 In the diagram, A, B, C, D, Elie on a
circle, centre 0. Angle AED = 110°.
·""-c
-~
\
Find
(a) LACD,
(b) LABD,
[Tum Over
Anglican High School Preliminary Examinations 03 2011 Math (4016/01)
6
11 A bag contains I 0 coloured balls of which 3 are grey, 5 are blue and 2 are red.
(a) Find the probability of picking either a grey or blue ball from the bag.
(b) When n grey balls are added into the bag. the probability of picking a grey ball
becomes 0.5. Find the value of n.
12 In a class of 40 students, x of them had a Sewing lesson and 18 had a Cooking lesson. Every
student needs to attend only one of the two lessons and 2 students were absent on that day.
(a) Draw a Venn diagram to illustrate the given information.
[I]
(b) From the diagram, find the number of pupils that had a Sewing lesson.
(Turn Over
Anglican High School Preliminary Examinations 03 2011 Math (4016/01)
8
7
13 Mr Tan's monthly savings (S) is directly proportional to the square root of his monthly
income(/). His income in January and February 2011 is $3 600 and $2 500 respective!}
His savings in January is $80 more than in February. Find the amount he saved in January
2011.
[3]
[Turn Over
Anglican High School Preliminary Examinations 03 201 I Math (40I6/01)
8
1.5 T'he follo,\ing table shows the amount of $2, $10 and $50 notes that Amy. Bala and Owen
have.
$2 $10 $50
Amy 3 I I
Bala 2 0 2
Owen 3 2 0
(c) Evaluate (1 1 1) M.
Answer (d) [I ]
(e) Write down, but do not evaluate, a product of three matrices which will give the total
amount of money that Amy, Bala and Owen have altogether.
(Turn Over
Anglican High School Preliminary Examinations 03 2011 Math (4016/01)
10
9
0 IO 20 30 40 50 60 70 80
Time (s)
(a) the acceleration for the first part of the journey,
(Turn Over
Anglican High School Preliminary Examinations 03 2011 Math (4016/01)
1
10
17 In a survey, a group of 14 people were asked the time taken to travel to work each day.
The time taken, recorded in minutes, are shown below.
31 22 40 55 11 29 37
28 29 37 46 14 29 35
The first column of the above data has already been entered on the stem and leaf diagram in
the answer space.
Complete the stem and leaf diagram and state the median.
2 2 8
[Turn Over
Anglican High School Preliminary Examinations 03 2011 Math (4016/0 l)
12
11
18 (a) The cwve y = k + 5x - x 2 , where k is a constant, cuts the x-axis at (1,0) and (4,0).
(ii) Hence, sketch the graph of the cwve on the answer space below. [2]
y
•( l, I)
0
x
(b) Sketch the graph of y = (x - 3)2 - 2 on the answer space below, indicating the line
of symmetry clearly. y [3)
•(I , I)
[Turn Over
Anglican High School Preliminary Examinations 03 2011 Math (4016/01)
13
12
B( _ __ _ _ _ )[l]
20 The diagram shows three circles with diameters 2r, 4r and 6r respectively. Find the ratio of
the area of the shaded region to that of the unshaded region.
14
13
[3]
(b) If the base area of the larger claypot is 72cm2, find the base area of the smaller claypot.
Answer (a)
[2]
[Turn Over
Anglican High School Preliminary Examinations 03 2011 Math (4016/01)
14
(b) What are the numbers in the S + 2 column commonly known as?
[Turn Over
Anglican High School Preliminary Examinations 03 2011 Math (4016/0 l)
16
15
24 (a) In the answer space below, draw a quadrilateral ABCD in which the base
AB= 10 cm, LBAD = 82°, AD= 7.2 cm, LABC = 105° and LADC = 75°. [3]
[Turn Over
Anglican High School Preliminary Examinations 03 2011 Math (4016/01)
Answer {b) The point P is equidistant from the lines ................. .
and .. .. .. .. .. .. .. .. .. .. and equidistant from the points
..................... and ........... ........... . [1]
I
E
~~ =~--~
(
- -- \ !
·-.-- -- ~~
'
\
'
I
2 0.01482 I
x
'
I
I
\
18
l
/
I
: .-
3 4a 3 b(a-3b)(a + 3b) - ---- - - --- '
(b) 20
4 (a) n =2 13 $480
pq3
5 $805 14 (a) e = -2
b-c
6 (a) 18m (b) a=--
d-e
[1~:]
7 (a) 15
l (a)
(b) x =--or
.., 2
..)
(b) The matrix states that Amy has $66, Bala has
8 O(x~2
$104 and Owen has $26.
9
(c) [8 3 3)
9 9th, 13th, 17th, 25th
(d) The matrix represents the total number of
-
[Turn Over
Anglican High School Preliminary Examinations 03 2011 Math (4016/0 l)
18
17
17 I I 4
2 2 8 9 9 9
3 I 5 7 7
4 0 6
5 5
18 (a)(i) k= -4
y
[Turn Over
Anglican High School Preliminary Examinations 03 2011 Math (4016/01)
18
-
y
\'
(b)
(~
..
(
v r
(3,-2)
(c) gradient of AB = - ~
2
20 Ratio= 4: 5
21 (a) 2:3
(b) Area= 32 cm2
22 ABCD = 9cm1
23 (a) 14, 16
23,25
(c) S = 79
(d) n2 =527 + 2
n =.J529 = 23
(e) n2 =S + 2
S:: n2 -2
(f) 10199
(Turn Over
Anglican High School Preliminary Examinations 03 2011 Math (4016/01)
20
19
24
END
Anglican High School Preliminary Examinations 03 2011 Math (4016/02)
Answer all the questions.
1. (a) In the diagram, the straight line ABC is parallel to ED and BF is parallel to CD.
LABF= 55°, LDEF= 115° and LBDC = 65°. State with reasons, the value of
A B c
(i) LBCD [I]
(b) One of the interior angles of a polygon is 140°. The remaining interior angles are each
equal to 160°. Find the number of sides of the polygon. [2]
2. The line of the equations ax +by= 3 and ex - dy = 5 intersect at the point (2, I).
(a) Show that 2a + b - 2c + d= -2. [3]
(b) If a =-c and b = 2d, show that 4c - 3d= 2. [2]
7 3-x
3. (a) Express as a single fraction - [3]
2(5-x) (x + tXx-5)"
2
(b) Using completing the square method, solve the equation x -3=5x. [3]
Anglican High School Prelim 3 Examinations 2011 Mathcma1ics Paper 2 [Tum Over
s
Anglican High School Preliminary Examinations 03 2011 Math (4016/02)
4. A car travelled from town X to town Y, which were 320 km apart, at an average speed of x
km/h. A van travelled by the same route from town X to town Y at an average speed which
was 5 km/h slower than the speed of the car.
(a) Write down an expression in tenns of x, for the time taken by the car to travel
from town X to town Y. [1]
(b) Write down an expression in tenns of x, for the time taken by the van to travel
from town X to town Y. [l]
(c) Given that the difference between the time taken is 15 minutes, fonn an equation
in x and show that it reduces to x 2 - 5x - 6400 =o. [3]
(d) Solve the equation x2 - Sx - 6400 =O, giving your answer correct to 2 decimal
places. [3]
(e) Hence, find in hours and minutes, the time it took the van to travel from town
Xto town Y. [1]
5. (a) A man deposited $5 000 in a bank at a certain simple interest rate per annum. After two
years, he found that his deposit with interest was $5 080.
(i) Calculate the simple interest rate that the bank pays per annum. [2]
(ii) If the man deposits the same amount of money in another bank which gives a
compound interest rate of 0.5% per annum, how much money, in dollars and cents,
will the man receive after three years. [2]
(b) The income tax to be paid from a man's earned income is calculated as follows:
the first $10 000 of earned income is tax free,
the next $10 000 is taxed at 3.5%,
the next $5 000 is taxed at 6%,
and the remainder is taxed at 8%.
(i) If the man earns $60 000 in a year, calculate his income tax. [2]
(ii) The man's wife pays a total of$ l 280 in income tax, how much does she earn in a
year. [3]
Anglican High School Prelim 3 Examinations 2011 Mathematics Paper 2 [Tum Over
Anglican High School Preliminary Examinations 03 2011 Math (4016/02)
6. (a) The diagram below shows a pyramid with a horizontal rectangular base ABCD and a
vertex Vvertically above A. If AB= 5cm, BC= ?cm ~nd VC = 20 cm.
Calculate,
(b) The diagram below shows a signboard suspended by three metal poles AB, AC and AD.
AC = 2.5 m and AD = 3 .5 m. Pole AC makes an angle of of 64° with the horizontal pole
AB. Calculate,
(c) Swimmers A and B left island X at the same time. Swimmer A swam 30 km on a bearing
of 130° from X and swimmer B swam 25 km on a bearing of285° from X. Calculate,
Anglican High School Prelim 3 Examinations 2011 Mathematics Paper 2 [Tum Over
Anglican High School Preliminary Examinations 03 2011 Math (4016/02)
7. The probability that Kennie leaves his house for school before 6.30am is 0.35. The
probability of him reaching school on time at 7.25am or earlier is 0.85. If he leaves his house
after 6.30am, then the probability of him reaching school late is 0.8.
(a) Draw a probability tree diagram to show all possible outcomes. [2]
(b) Find the probability that Kennie will be late for school. [I]
8. In the diagram, t:.ABC is an equilateral triangle which has a circle inscribed in it. Wand X are
points of tangency on the circle. WY and XZ are the diameters of the circle.
(a) Find, (
9. The cumulative frequency curve below illustrates the daily wages of 76 workers in Factory A.
Number of workers
Factory A
80 ..........,..............,...................................................................................................................................................,..,..,l"'T"I"...............................................,..,..,~
c so
=
~
=
c:r
~
""'
~
~ 40
>
:c
cs
=6=
u 30
0 10 20 30 40 so 60 70 80
Anglican High School Prelim 3 Examinations 2011 Mathematics Paper 2 [Tum Over
Anglican High School Preliminary Examinations 03 2011 Math (4016/02)
9. (b) (i) Copy and complete the grouped frequency table of the daily wages of the workers in
Factory A. [2]
(ii) Show that the estimated mean of daily wages is approximately $37.61, correct
to 2 decimal places. [ 1]
9. (c) The daily wages of another group of 76 workers in Factory Bis illustrated by the box and
whisker diagram below.
Factory B
I I I I I
~ ~+ l- .•
0 10 20 30 40 50 60 70 80
Daily wages ($)
Compare and comment on the daily wages of the two groups of workers. [2]
Anglican High School Preliminary Examinations 03 2011 Math (4016/02)
- -
10. In the diagram, OABC is a parallelogram and Xis the midpoint of BC. OC is produced to D
such that OC =CD. OA = 4p and AB= 2q.
(i) OB
(ii) AX
-- [l]
[l]
(iii) BJj [1]
Anglican High School Prelim 3 Examinations 2011 Mathematics Paper 2 {Turn Over
Anglican High School Preliminary Examinations 03 2011 Math (4016/02)
11. A toy manufacturer cuts off a top part of a hollow cone to make a frustum so that a sphere
can be placed on top of it as shown in Figure 2.
Figure I Figure 2
Figure 3 shows the cross-section of the object cut vertically through the centre. The circle
with centre 0 passes through points A, Band C. OA is produced such that OA =AX and OC is
produced such that OC =CY. The radius of the circle is 8 cm and LAOC = f( rad .
3
,, ,,0... ...
,, ,, ... ...
,, ,, ' ...
Figure 3
Calculate,
Anglican High School Prclim 3 Examinations 2011 Mather.iatics Paper 2 (Tum Over
Anglican High School Preliminary Examinations 03 2011 Math (4016/02)
12. Answer the whole ofthis question on a sheet of graph paper.
The variables x and y are connected by the equation y = _!_ x 2 (3- x).
2
Some corresponding values of x and y, corrected to 1 decimal place where necessary, are
given in the following table.
(a) Using a scale of 2 cm to 1 unit on each axis, draw a horizontal x -axis for
-1.5 ~x~3.5 and a verticaJy-axis for -3.5 ~y ~ 5.5. On your axes,
plot the points given in the table and join them in a smooth curve. [3]
(b) By drawing a tangent, find the gradient of the curve at the point (2.5, 1.6). [2]
(c) Use your graph to find the largest solution of _!_ x 2 (3 - x) == 1 . [I]
2
(d) On the same axes, draw the graph of 3y + 2x == 6 for -1.5 ~ x ~ 3.5. [l]
(i) Write down thex-coordinates of the points where the two graphs intersect. (1]
END OF PAPER
A::zfican High School Prelim 3 Examinations 2011 Mathematics Paper 2 [Tum Over
Anglican High School Preliminary Examinations 03 2011 Math (4016/02)
(iv) LBFE = 120° (c) (ii) bearing of A from Bis 118.7° (ldp)
(b) 17
2 N.A 7
O~to school
-s:
before 6.30 a.a:i
Oot to school
(a) 2(x + ()(5 - x) late
4
(b) x =5.54 or -0.541 s
Leave home
after 6.30 am
~-
o~
.
Oct to school
late
to ac;hool
(a) 320
x (b) P (kennie will be late)= 0.5725.
(b) 320
x-5
(c) N.A
Anglican High School Prelim 3 Examinations 2011 Mathematics Paper 2 [Tum Over
32
Anglican High School Preliminary Examinations 032011 Math (4016/02)
C.)1
Ill i
CANDIDATE
NAME ( ) 4/---
CENTRE INDEX
NUMBER I I I I I NUMBER I I
MATHEMATICS 4016/01
Paper1 15 August 2011
2 hours
Candidates answer on the Question Paper.
Write your Centre number, index number and name on all the work you hand in.
Write in dark blue or black pen on both sides of the paper.
You may use a pencil for any diagrams or graphs.
Do not use staples, paper clips, highlighters. glue or correction fluid.
At the end of the examination, fasten all your work securely together.
The number of marks is given in brackets [ ] at the end of each question or part question.
The total number of marks for this paper is 80.
[Turn over
Mathematical Formulae
Compound interest
Mensuration
1
Volume of a cone= - 1fr 1 h
3
4
Volume of a sphere= - 1fr 1
3
Trigonometry
a b c
--=--=--
sin A sin B sin C
a 2 =b 2 +c 2 -2bccosA
Statistics
Mean= 'L.fx
'LI
Standard deviation =
'L.fxl
- -- -- (I.Jr:li
Lf I.!
1
1 (a) Express 2 - as a percentage.
40
(b) [1]
. . 5 -4x x-3
2 (a) SoIvet he mequa 11ty ----.S- ~ - - + 1
2
(b) l lence write doVv1l the smallest perfect square which satisfies
5-4x x-3
--~--+I.
5 2
(b) [1]
(a) x 2 -16a 2 ,
(b) [2]
(b) n= -- - (1)
S A polygon has n sides. Two of its exterior angles are 90° and 60°, and all
the remaining interior angles are 166° each. Calculate the value of n.
Answer n= (2J
Us1 Us.
6 (a) Given thatx = 7 is a root of the quadratic equation x 2 +kx= 14,
find the other root of the equation.
2 3
- + -= -9,
x y
.!.- s=.!..
x y
{b) x=
y= [21
(- 1, -7)
Answer y= [2}
(b) Find an expression, in terms of n, for the nth term of the sequence.
(c) Explain why the number 50 285 is not a term of the sequence.
Find
(a) the time, in minutes, the triathlete takes to swim the 1 500m,
(b) the average speed for the total distance of the race.
(b) Find the smallest integer n such that 4 200n is a perfect square.
(c) Written as products of their prime factors, the HCF and LCM of the
numbers 4 200 and x are given below.
(b) ll = [l]
(c) - - - - - - - - [l]
Cedar Girls' Secondnry School 1 1016/0l /S4/ Prclim 2/201 I (Turn over
:s
8
For For
Ezammtr's ~mine
Un Use
11 A nature reserve of area 225 km2 is represented on a map by an area of
36 cm2 •
Find
(b) the area of the nature reserve, in cm2, on another map whose scale is
l cm to 5 km.
(b) nwn ll I
[1]
[I]
35 -· - - - -
30
25
20
15
10
5
(a) Express the sales of Brand Bas a percentage of the total sales.
(b) The same information is shown on a pie chart. Find the ang1e of
the sector which represents the sales of Brand D.
(c) The advertisement made the claim that 'Brand C is the most
popular soft drink in Singapore in 20 11 '.
Is the claim valid? Give a reason to support your ans\.ver.
[2]
Use the information in the article and the table above to answer the questions.
(a) Calculate the total selling price of all apartments in The Sapphire.
Give your answer in standard form.
(b) Find the exact difference in the total selling price of all apartments in The
Sapphire and the total selling price of all apartments in The Emerald.
Give your answer in million dollars.
(c) The selling price of a studio apartment in The Garnet is 110% more than
the selling price of a studio apartment in The Sapphire.
Find x, the selling price per square foot for an apartment in The Garner.
(c} x = [2]
Number of hours 0 1 2 3 4 5 6
Number of pupils 4 x 7 6 5 4 2
(c) If the mean number of hours spent by each pupil is 2.75, find the value
of x.
(b) [I]
(c) [2]
17 In the figure, the area of Lriangk OA Bis 8 square units and OA OB.
y
x
0 A
Find
(a) e.
(b) the coordinates of A and R.
0
;f nswer (a) 0 =- [l]
(b) A( _ _ , _ ) [I]
IJ( _ _ , _ ) [I ]
(c} l 1J
hem
(a) If the ratio of the capacity of the hemisphere to the capacity of the
cylinder is I : 4, show that h = ~ r .
3
{b) If the total capacity of the container is SO.?r cm3, find the radius r.
3
(c) Find the area of the metal sheet used to make the container.
Leave your answer in terms of 1T.
Answer (a)
(1 J
(a) Find, in the simplest integer form, the ratio of the length of the
smaller container to the larger container.
(b) The base area of the smaller container is 9 cm2• Find the base area
of the larger container.
A .....___
D--------~ C
Answer (a)
[21
Answer (hJ [I J
__.-- - ··--..-......
Time [2]
Find
North
[31
Distance (m)
200
0 20 35 50 Time (t seconds)
(a) Find
(i) v,
(b) On the grid in the answer space, draw the speed-time graph of the same
journey.
(b) (2]
Speed (mis)
0 20 35 50 Time (t seconds)
End of Paper
PAPER 4016/1
l(a) 202.5 % 13(b)(ii) Mr.N={2}
3 13(b)(iii) 7
l(b) -
14 14(a) 25 %
2 14(b) 67.5°
2(a) x;::: l -
13 The claim is not valid
2(b) 4 - Small sample size (data taken
3(a) (x + 4a)(x - 4a) 14(c) from only one secondary school
- Data based on only one day (not
3(b) (x-4a)(x + 4a-2y) representative of the entire year)
2b
4(a) - lS(a) $( 2.672 x I 0 8 )
3ac
4(b) 4 15(b) 33.53 million dollars
5 17 15(c) I 400
6(a) -2 16(a) O~x~6
I 1 16(b) 5
6(b) x=- and y=--
3 5 16(c) 4
2
7 y=3x +6x-4 17(a) 45°
8(a) 162 and 486 l 7(b) A ( 4, 0 ) B ( 0. 4 )
8(b) 2 (3) 111
- 17(c) y=-x+4
18(b) 2cm
It is not a multiple of 21
8(c) I
It is not an even number 18(c) 29-1l' cm2
3
9(a) 16~ minutes
19(a) 2:5
3 19(b) 56.25 cm2
9(b) 14.8 krn/b AD =DC (sides of rhombus)
lO(a) 2 x3x5 2 x7
3 LADB = LCDB( diagonal of
lO(b) n= 42 20(a) rhombus bisects LADC)
DL is common side
lO(c) 2 2 x3 3 x5xll :. tMLD = LJCLD (SAS)
ll(a) 1 : 250 000 20(b) 90°
ll(b) 9cmt Depth
12(a) 56.25 %
12(b) 80 men
'm:J
21
13(a)
Time
'W 9G
22(a) 45.9 cm
p
13(b)(i) ~cp 22(b) 147 cm2
-
PAPER 4016/1
l(a) 202.5 % 13(b)(ii) Mr1N={2}
3 13(b)(iii) 7
l(b) -
14 14(a) 25%
2 14(b) 67.5°
2(a) x~ l -
13 The claim is not valid
2(b) 4 - Small sample size (data taken
3(a) (x + 4a)(x -4a) 14(c) from only one secondary school
- Data based on only one day (not
3(b) (x-4a)(x + 4a-2y) representative of the entire year)
2b
"(a) - lS(a) $(2.672x 10 8 )
3ac
4(b) 4 15(b) 33.53 mill ion dollars
5 17 lS(c) 1 400
6(a) -2 16(a) O~x~6
I 1 16(b) 5
6(b) x=- and y= - -
3 5 16(c) 4
2
7 y = 3x +6x-4 17(a) 45°
8(a) 162 and 486 l 7(b) A ( 4, 0 ), B ( 0, 4 )
8(b) 2(3)"-I 17(c) y=-x+4
18(b) 2cm
It is not a multiple of 2 /
8(c) I
It is not an even number 18(c) 29-.1l" cm2
3
9(a) 1 6~ minutes
19(a) 2:5
3 19(b) 56.25 cm2
9(b) 14.8 km/h AD =DC (sides of rhombus)
lO(a) 2 3 x3x5 2 x7 LADB = LCDB( diagonal of
lO(b) n =42 20(a) rhombus bisects LADC)
DL is common side
lO(c) 2 2 x3 3 x5xl l : . MLD = LJCLD (SAS)
ll(a) 1 : 250 000 20(b) 90°
ll(b) 9cmJ. Depth
12(a) 56.25 %
J2(b) 80 men
'm:J
21
13(a)
T ime
13(b)(i)
w /:J
f,
s 6 7
p
22(a)
22(b)
23(c)
45.9 cm
147 cm2
6.2 ± 0.1 cm
CEDAR GIRLS' SECONDARY SCHOOL
Preliminary Examination 2
Secondary Four
MATHEMATICS 4016/02
Paper 2 16 August 2011
2 hours 30 minutes
Additional Materials: Answer Paper
Graph paper (1 sheet)
Write your Centre number. index number and name on all the work you hand in.
Write in dark blue or black pen on both sides of the paper.
You may use a pencil for any diagrams or graphs.
Do not use staples, paper clips, highlighters, glue or correction fluid.
At the end of the examination, fasten all your work securely together.
The number of marks is given in brackets [ ] at the end of each question or part question.
The tot al n umber of marks for this paper is 100.
[Turn over
3
Mathematical Formulae
Compound interest
Mensuration
4
Volume of a sphere - -;rr 1
3
Trigonometry
a b c
--=--=--
sin A sin B sin C
a2 =b
2
+ c2 - 2bc cos A
Sratistics
l. ·fr·
Mean=-
l.f
L:fx
Standard deviation = - --
Lf
2
('-I./x)
-
2..f
2
3
3
1 ( a) .
G iven th at y = k - x ,
•
express x m tenns o f y, k and 1r. [2]
3
7CX +1
1 3
(b) Express as a single fraction in its simplest form - [2]
2(x-2) 2 x(x-2)
(ii) Hence solve the equation x 2 -~x-5 = 0, giving your answers correct to
4
two decimal places. [3]
2 Mr Goh bought a home entertainment system during the Year-End Sale on hire
purchase. He paid a deposit of 20% of the selling price. The balance of the payment
was $8 000.
(b) Mr Goh paid the balance of the payment in monthly instalments of $380,
charged at x % per annwn simple interest for 2 years. Find the value of x. [3]
(c) Mr Goh could have taken a loan to pay the outstanding balance. lf the finance
company charged him a compound interest of 6.5 % per annum with repayment
period of 2 years, would you recommend that he takes up the loan instead of
paying in instalments? State your reason clearly. [3]
3
M
Three points, A, Band C, lie on a horizontal field. A is due North of Band the
bearing of C from A is 076°. M is vertically above A. Angle J,/BA = 43°, AB = 12 m
and AC 20 m.
(a ) Calculate
(b) P is a point on BC such that the nngle of elevation of M from P is the greatest.
Calculate the angle of elevation of \f from P. [3 J
(a) Write down an expression, in terms of x, for the number of litres of petrol used
when the car is driven on rough terrain for 400 km. [ 1]
(b) Write down an expression, in terms of x, for the number of litres of petrol used
when the car is driven on an expressway for 400 km. [l]
(c) The car uses 5 litres less of petrol when it is driven on an expressway for
400 km compared to when it is driven on rough terrain. Write down an
equation in x to represent this information, and show that it reduces to
x 2 + 2x - 160 = 0 . (3]
(d) Solve the equation x 2 + 2x- l60 = 0, giving both answers correct to
two decimal places. [3]
(e) Calculate the number of litres of petrol used when the car is driven on an
expressway for 100 km . [2)
(iii) Given that A is the point ( - 5, 2), find the coordinates of B. [l]
(b)
p
(a) RS, [ l]
(ii) Write down two facts about the points T, U and Q. [2)
area of triangleRUQ
(b) area of triangle PQS [2]
6
A
In the diagram, DE is a tangent to the circle \Vilh centre 0. ABC and AOF are
straight Jines and L.CBE = 61°.
(b) Are the lines AE and DC parallel? Give a reason for your answer. (2)
7 (a) A four-sided unbiased die and a six-sided unbiased die are thrown at the same
time. The score of each die is noted.
(i) Draw a possibility diagram to show the outcomes of the throw. (1]
(i) Draw a tree diagram to show the probabilities of the possible outcomes. (2]
(a) the two cups of ice-cream selected are of the same flavour, [2]
8 Ashley, Ben and Celia are salespt:rsons at a travel agency. The sales thal they
obtained for each of the three tour packages in May and June are shown in the table
below.
May June
Ashley Ben Celia Ashley Ben Celia
Package 1 5 8 6 9 14 15
Package 2 7 4 5 13 10 9
Package 3 4· 3 5 12 11 10
The infonnation for the monlh of May can be represented by the matrix
M= 7
5 48 5.
6)
(4 1 5
(a) The information for the monlh of June is represented by the matrix J .
(ii) Evaluate J - M
Stale \\.hat Lhe clements of J - M represent. [2]
(b) Write down an appropriate matrix X and use it to evaluate the matrix product
M X ~hich gives the total number of sales of each package for the month of
May. 12]
(c) The prices of packages I. 2 and 3 are $1 000. $1 500 and$ 3 000 respecti\ ely.
The sales commission for each of the packages is 8% of the price.
This information can be represented by a row matrix C.
Cedar Girls' Sccondal) "ichool -1016 021S.t Prclirn 212011 (Turn over
31
10
9 The cumulative frequency graph below shows the distribution of marks of 300
students from School A in an English examination.
Cumulative Frequency
. ~__;__ I
~--
.~ .. +
r - - - ---
+ • +
--- I ~t:=~~I -1 .- - -
____:!__vv . - . . . - -·-j--1· -
- I . +
~
>-
-.·-·
- - --
..
--"j"-
.. . ·---
~!. .- . -
._ . I·. - -
..
-... . .
- -----
-
+
,_
~,.;e ,,
--. f--,----
-
~
-- p::::.:: --·-
-.-..- -"~""
-- ..
• + +
.. . -- i--.-·-
. I
I - -- ·--·- -~-
- --. --
+-
- -. . ...
I
+
.-
- - -- .--
- -
[
.._ -
H-00 - -
- ...
.
1
..__ - ~
--
- - ,.·- ~ ~
I
. -1
/- -- - ... ~
I
-1 -
•
- -l
I
- 1--50 -- --- . - .
-
.
I
I './ - --- ·-
..
- f--
-
._ + ..
Ii -
-YOO - --
J
f
.
t
I I .
.
.. .
.
I
' ..
--- --- - · -
---:n.; -
~--1
I
/.
Y· i
-
.
- +
..
.
I
I
.
.
--
t.
i.
..• I•
I
- - --
-1 +
.. . ...--·
+
.
•
~ ...- .
......... r
~ ..
t +
'
+ ..
' t -- -- --
L---"" ' I
i
I 0 ---- '0
.
1-· "'a rn .
. H)
I
1no • +
Marks
I
--
(a) Find the median mark. (1)
(c) The top 20% of the students obtain an Al grade in the examination. Find the
lowest mark a student must score to obtain an Al grade. [ ll
300 students from School B took the same examination. The table below shows the
information for this school.
Median 65
Interquartile range 37
(d) Compare briefly, the results for the two schools. (2)
(e) Copy an~ complete the grouped frequency table of the marks scored by the 300
students m School A.
Marks 0 <x $ 20 20 <x $ 40 40 <x < 60 60 <x < 80 80 <x < 100
Frequency 30 125 [2]
(f) Using your grouped frequency table, calculate an estimate of
12
The table below gives some values of x and the corresponding values of y.
(b) Using a scale of 1 cm to represent 0.1 unit, draw a horizontal x-axis for
0.3 $ x $ 1.6.
Using a scale of 1 cm to represent I unit, draw a vertical y-axis for -10 Sy S 5.
On your axes, plot the points given in the table and join them with a smooth
curve. [3]
(c) Use your graph to find the value of x in the range 0.3 $ x $ 1.6 for which
l
x-- < - 8. [2]
x2
(d) Find the x-coordinate of the point on the curve at which the gradient of the
tangent is 3. [2]
(e) (i) On the same axes, draw the graph of y = -5x + 4. (1]
(ii) Write down the x-coordinate of the point at which the two graphs
intersect. [ 1]
1
(iii) Find the equation, in the form - 6x + ax 2 + b = 0, which is satisfied
by the value of x found in (e)(ii). (2]
End of Paper
CEDAR GIRLS' SECONDARY SCHOOL
SECONDARY 4 MATHEMATICS
Answer Key for 2011 Preliminary Examination 2
Mathematics 4016/2
Sbic - 3 1
x=v~-:;
la UQ= - - a--b
2 2
lb 12-Sx Sb id -
TQ = -3 a - b
2x(x-2) 2
Sbii - = -1 TQ
- . T, U and Q are collinear
(x -~)'
lei UQ
_5_2_ 2
8 64
U is the midooint of TQ
lcii x = 2.64' x = -1.89 (2 d.p.) Sb iii a 3
-
4
2a $10 000 Sbiiib l
-
12
2b x=7 6ai L.AOE = 122°
2c Total amount after interest from 6aii LAFE= 6 1°
finance company= $9 073.80 6aiii LAEO- 29°
Yes. l [e should take up the loan from 6aiv LAED = 61°
the finance company as the interest is 6b Yes. LEDC = LAED (alternate angles,
lower. AE and DC are oarallel).
3ai .\l-1 l l.2 rn 7ai
Sa ii 5 7hiia 17
- -
3 40
Sa iii B(-2,- 3) 7biib 37
-
-
80
Sbia --
RS =2a
Sui 9 14
.._
J [1 3 10 1:]
Sbib -RP 12 11 10
'~' t ~h
8aii 9c 81 marks
J-M=(: : :J
8 8 5
9d Pupils from school A performed better
as their median is higher. The results o~
The elements represent the difference pupils from school A are more
in the number of sales by each consistent as the interquartile range
salesperson between the months of is lower.
June and Mav for each package.
Sb 9fi Mean = 64.3 marks
9fii Standard Deviation = 21.9 marks
lOa p = 0.82, q = 3.71
Sci C=(80 120 240) lOc Drawy =- 5
0.3::;; x < 0.35 (± 0.02)
8cii CM = (2200 1840 2280) lOd x=l
The elements represent the sales lOeii x = 0.61(± 0.02)
commission each salesperson
received in the month of May. lOeiii -6x 3 + x 2 + 1 = 0
9a 70 marks I
9b 27 marks
7bi
I Ob -
'
: . ·' • , ; ~ ;
! : : T. :· : ; .
: ! :
r: ....
: : :
;· : : t
:. •
I
·· i . '
! ;:
: :· ·:
:
..
~~
'.; ~~~l:j.~i ,: · I :
t 2 • i , · : · .' , : ·1 • • ,Y ·' 3 +,· x :..,..i. : . .
~- :: ~~ :
1
i c : <s ,, :; A- ·'
~ . ____________.:- ~ . 7~ ---·__ : . :~ _: ~ .
: io o. 1 · o. a ~ o.l .;·•r
; o. :;> ~ o. e 0. 1 • o. e ' ~ 1. 1 "' 1.t · • > ' · " : i..s 1. 1
t -2 • ' ~ ~:· ~ ~-
: ; : ~ ~ : ~ t : ~ ~ Y. = -?x +~ 4
: _: : : ~ ~ = : • :
::::t:~~ .:::;:·::~. :::j:::~:::r::;:::::;··:.~::.:j::.·;::::;: : ~::-:'. .:;~ ~~:·~::::(:i::::;:: :i. . ;: .: ~ .:::t::.[•.::I :..,...·;·: :;: : :~.···~· ·::::::.!:::.~::::
MATHEMATICS 4016/01
Paper 1
15AUG2011
Candidates answer on the Question Paper 2 hours
Write your name and register number on all the work you hand in.
Write in dark blue or black pen.
You may use a pencil for any diagram~ or graphs.
Do not use staples, paper clips, highlighters, glue or correction fluid.
rTnrn nv"r
3
3 S.8+84.07
(a)
7.0SX7.003
(b) (7.13 x 103 )-:- (27.l x 10- 4 - 0.147 x 10- 2 ). Give your answer in standard form.
2 Two maps are drawn of a new town. On the first map, a school appears with an area of 3 cm2 and on the
second map with an area of 12 cm2 . Given that the scale of the first map is 1:80 000, find the scale of the
second map in the form I :n.
(Turn over
CGS Prehm Exam 2011 4016/01
4
3 A company charges $530.40 for transporting goods weighing 48 kg for 36 km. If the charge is 35cents
per kg per km for the first Skm, how much less is the charge per kg per km for every subsequent km?
4 The diagram shows a quadrant of a circle, centre 0 and radius 28 cm.Mis the midpoint of OB and a
semi-circle is drawn with MB as diameter. The other semi-circle in the quadrant has diameter OA.
Find the perimeter of the shaded region. Give your answer in the form a+ brr.
28
3E
5
5 (a) Find the range of values of x for which 1 - 3x ;:::: 4x - 2 and -2x < 7 and then represent the
solution set on the number line.
(1]
y = ................................. [3]
(Turn over
(GS Prehm Exam 2011 4016101
6
~<.>
.....
0
...
0
.0
§
z
During a traffic survey, the number of people in each car passing through a checkpoint was noted.
The results of the survey are shown in the bar chart.
(a) Explain why this bar chart might be considered misleading.
(b) Express the total number of cars which had at least 2 people as a fraction of the total number of cars.
Give your answer in its lowest terms.
(c) The same information is to be shown in a pie chart.
Find the angle of the sector which represents cars with three people.
......................... ......... ......... ..... .. ... ........... .............. ... ................. ............. (1)
It
7
- A flash drive which Wendy bought has 20 gigabytes of memory space. The average size of a digital file
is 4.5 megabytes.
(a) 20 gigabytes can be written as k trilllon bytes. Find k.
{b) Find the number of complete digital files the flash drive can store.
(c) Given that 680 digital files have been stored, find the size of the free memory space in bytes. Give
your answer in standard form.
9 The braking distance of a car is directly proportional to the square of its speed.
When the speed is p metres per second, the braking distance is 6 m.
When the speed is increased by 250%, find
(a) an expression for the speed of the car,
(b) the braking distance,
(c) the percentage increase in the braking distance.
11 The vertices of triangle PQR are (-2, -2), (2, 4) and (2, 1) respectively.Sis the point (2, -2) and
Tis the point (2, 6).
(a) Find the length of PQ.
(b) Find the area of t:.PQR.
(c) Expressing your answer as a fraction in its lowest terms, write down the values of
(i) tanLPQT,
(ii) COSLPRQ.
'I
T
e ><
4 Q
3 4 5 x
p
-3
Answer (a) ........................ units [I]
............................................... (1)
[Turn over
10
13 The following table shows the nutrition contents (in grams) of a packet of milk and a serving of
instant noodles.
Protein Fat Carbohydrate
Milk 6 7 10
Noodles 11 22 59
. . . . ... . . . . . . . . . . . . . . . . . . . . . . . . . . . . . . . . . . . . . . . . . . . . . . . . . . . . . . . . . . . . . . . . . . . . . . . . . . . . .. . . . . . . . .. . . . . . . . . . . . . . .. . . . . . . . . . . .. [l]
16 In the diagram. ABCD is a parallelogram. The AT:TD = l :2 and BTP is a straight line. CT meets BD at
point Q and is parallel to DP. Find the value of
Area of APTD
(a)
Area of ATBC .·
Area of llPTD
(b)
Area of ADTC
TQ ;;
(c)
CQ
[Turn over
12
17 The diagram is the distance-time graph for the first 25 seconds of a car's journey. The car started from
rest.
_J
Distance
400 f-
(metres) l
I
10 15 2.5
'
. J
Time (t seconds)
30
1 t
-i
Speed
(metres per second) 20 .. !
I' I
I
!
~o
I' I I I
I
i -t t
..
'
0 5 1 10 15 2P 25
Time (t seconds)
(2)
Two similar right pyramids shown below, A and B, have base areas of 28 cm2 and 63 cm2 .
The density of pyramid A is thrice the density of pyramid B.
(a) Find, in its simplest integer form, the ratio of the height of pyramid A to the height of pyramid B.
(b) The total surface area of the triangular faces of pyramid A is 68 cm 2 •
Find the total surface area of the triangular faces of pyramid B.
(c) The volume of pyramid A is 336 cm3, find
(i) the height of pyramid A,
(ii) the volume of pyramid B.
(d) The mass of pyramid Bis 189 g. Find the mass of pyramid A.
Pyramid A
Pyramid B
[Turn over
14
19 Town X and Town Y are 36 km apart. A cyclist left Town X for Town Y at 08 00, cycling at 16 km/h.
A motorist left Town Y for Town X at the same time as the cyclist, travelling at 24 km/h.
(a) Draw the distance-time graphs of the cyclist and the motorist on the same axes.
(b) Use your graph to find
(i) the time the cyclist passed the motorist,
(ii) the distance from Town Y when the cyclist passed the motorist,
(iii) how much longer the cyclist took to complete the journey compared to the motorist,
(iv) the time when the cyclist and the motorist were 26 km apart.
(iv)................................ [1]
... The scale drawing in the answer space below shows the positions of three shops A, Band C. A is due
North of B.
(b) On the map, the shop Dis 6 cm from Con a bearing of057°. Find and label the position of shop D.
(c) A new shop Eis to be built equidistant from AB, BC and CD. By constructing bisectors, find and
label the position of the new shop E.
(d) Given that BC= 1600 m, calculate the actual distance from D to the new shop E .
[3]
D
c
4p + 15q
A 16p 8
Answer {a) (i) Bi5 = .. ... ...... ....... ... ..... [I)
[2]
(d} ................. ............ cm 2 [I]
COS Prehm Exam 2011 4016/01
er Key 4 2 2
16 (a) - (b) - (c) -
9 3 3
I a) 1.22 (b) 5.75 x 106 17 (a) 20 (b) 2
- l : 40 000
5
14 + 35rr
1
(a) -3-
2
< x <- -37
·1
i 1 t• "l l h l
·2 ·1 0 .) 1
·3.5 18 (a) 2:3 (b) 153 (c)(i) 36 (ii) 1134 (d) 168
1"
I 'q
(b) X = 16 y = -5 I
·~· ·- -f 1
(a) The bar chart might be considered
misleading as the vertical axis does not start
from zero. Hence the proportion of the length of
the chart is not accurate.
19 (a)
(b) £20 (c) 144°
(b) (i) 0854 (ii) 21.5 (iii) 45 (iv) 08 15
- (a) 0.02 (b) 4444 (c) I.694xl0 10 20 (a) 317° (d) 1429 i
' (a) 15° (b)24 (c) 165°
(a) 3.5p (b) 73.5 (c) 1125% (b) , (c)
0 (a) 158, median height= 165.5 cm
(b) x ~ 170
3 Two friends John and James recorded their time travelled from Town A to Town B.
The cumulative frequency curve below shows the time travelled by James on 50 trips .
• ~ ! I. ~ ili:
~++ - -~~ ~.L~•.;:~-+,_,.+..,..,__-t-
..........i---t----""'i-1-'
(i) Copy and complete the grouped frequency table of the time travelled by James. (2]
Times, x (min) frequency,/
130 <x ~ 135
135 < x ~140
4 A trader bought some paraffin for $400. He paid $x for each litre of paraffin.
(a) Find, in terms of x,
(i) the number of litres of paraffin he bought, [1]
(ii) the selling price per litre, if he lost 3 litres of paraffin and sold the remainder
of the paraffin for 80 cents per litre more than he paid for it. [l]
(b) He made a profit of $36. Write down an equation in x to represent the information,
and show that it reduces to 15x 2 +192 x-1600 =0. (3]
(c) Solve the equation t 5 x 2 + 192 x - 1600 = o, giving your answers correct to two
decimal places. [2]
(d) Find, correct to whole number, the number oflitres of paraffin he bought. [l]
n x z y
(a) (i) Draw the next expected pattern for Fig.4. [1]
(ii) Find the values of u, v and w. [3]
(b) Find the number of small right-angled triangles in Fig.5. [l]
(c) Find the total number of dots needed to form Fig.IO. [I]
(d) Express x, y and z in terms of n respectively. [3]
5
6
6 In the diagram, TA and TB are tangents to the circle at the points A and B respectively.
TBS is a straight line. The straight lines AB and QP intersect at Y. LQAB = 27°,
LPBT = LPQB = 40° and LATB = 56°.
~~~==-~~.C::::::..~~~~~~~~-'-~~r
B
Calculate and state the reasons clearly [1]
(a) L PAY, [l]
(b) L QBP, [1]
(c) L TAP, [2]
(d) LAYP, [2]
(e) LACQ. [2]
7 (a) Mr Fong left behind $252 000 to his wife and three sons. His wife got 35% of the
money and the rest was to be shared among his children, Ann, May and Chris, such
that Ann got two fifth as much as Chris and Chris got thrice as much as May.
How much did Chris get? (3]
(b) May bought a book on-line for S$52.20 which included a Value Added Tax(VA1)
of7.5%. The exchange rate then was £1 = S$2.02, calculate the VAT paid in
Pounds. [2]
(c) Ann bought a multi-function colour printer by paying zero downpayment and monthly
instalments of $177. 75 over 24 months. If the cash price of the colour printer was
$3950, find the rate of the interest per annum. [3]
(d) Chris puts all his money in fixed deposit with a bank that offered a compound interest
of 2.4% per annum compounded half yearly. Calculate the amount he would have at
the end of three years. [2)
E
E. F and Gare points on level ground as shown in the diagram. EG = I 02.5 m, FG = 80 m,
LEFG = 112° and bearing of F from Eis 312°. A vertical clock tower GT stands at G, a man
walks aJong EH. Calculate
(a) (i) the bearing of F from G, [2]
(ii) the bearing of E from G. [4]
(b) Given further that angle of depression of F from Tis 35°, find
(i) the height of the clock tower, [I]
(ii) the greatest angle of elevation of the top of the clock tower from the man's view
point and how far he has walked from his starting point E to this view point. (3]
9 (a) c
In the above figure, the sector CAB has centre C and radius 8 cm. CD bisects LA CB
and 0 is the midpoint of CD. An arc, centre 0, LEOF = 2tr and radius 4 cm is drawn
5
to meet CA and CB at E and F respectively.
(ii) Find the area of the shaded region ADBFE, correct to 2 significant figures. [3]
Q
(b)
8m
Sm B
10
l.6m
A container which is made by a square pyramid of height 1.2 m and a cylinder of radius
2 m and height 1.6 m is shown in the diagram above. The vertices of the square base of
(a) Prove that the length of the square base of the pyramid is 2.J2 m. [2]
(c) Calculate the total surface area of the container, correct to one decimal place. (4]
2 3
y=x +--5.
x
The table below shows some corresponding values of x and y. The values of y are given correct
x
y
I o.5
1.3
1
-1
1.5
k
2
0.5
2.5
2.5
3
5
4
11.5
5
20.6
6
31.5
Using a scale of2 cm to 5 units, draw a vertical y-axis for -10 $ y $ 40.
On your axes, plot the points given in the table and join them with a smooth curve. [3]
(c) Find the x-coordinate of a point on the curve with a gradient 5. [2]
( d) Showing your method clearly, use your graph to find the value of x in the range 2 $ x $ 6
for [1]
3
2
x +--30 =o.
x
(e) On the same axes, draw the graph of the straight line y = ~x - 4. [I]
(f) (i) State the x-coordinate of the points at which the two graphs intersect. [2]
(ii) Find the equation, in the form 2x3 + ax2 + bx + c =-0, which is satisfied by the values
End of Paper
·•·..
DUNMAN HIGH SCHOOL
Preliminary Examination
• 0
Year 4 SAP
Mathematics PAPER 1
4016/01 15 Sep 2011
2 hours
At the end of the examination, fasten all your work securely together.
The number of marks is given in brackets [ ] at the end of each question or part question.
The total number of marks for this paper is 80.
I
This document consists of 17 printed pages and 1 blank page.
3
[Turn over
© rJHS ?011
3
l Evaluate
(1
~Jx0.2-12
(a) 3
to 1 decimal place.
10
(b) [2]
DHS 2011 Preliminary Sec 4 SAP Mathematics Paper 1(4016101) (Turn over
e OHS 2011
4
3
. i·i:..
Stmp IJ.y°
V- sa b x o.sa- b
3
/,
2 2
2avb
Answer (3]
4 The distribution of the teachers' age group in Dunman High School is shown in the table.
(a) Calculate the mean age of the teachers.
(b) The distribution is to be shown in a pie chart. Calculate the angle representing the age grou
of 40 ~ x < 50.
30 s x < 40 46
40 s x < 50 22
50 s x < 60 10
(b) 0
[l
Mr Kwong, who runs an IT shop, bought some ipads at $x each. He intends to sell each ipad at
double the cost price. However, sales has been poor and he decides to give a 20% discount.
Calculate his profit percentage for each ipad sold.
Alex deposited $.54,000 at LHL bank which pays an interest of 4.8% per annum. Given further that
the interest is compounded monthly, calculate the amount of money he will get after 3 years.
[Turn over
6
Answer [3]
(a) Given that 24n is a perfect cube. where n is a non-zero integer, find the smallest possible
value of n.
JO Two similar Pepperoni pizza slices are sold in the Pizza Restaurant. The radii and the prices of the
pizza slices are as shown below, which pizza is more value for money? Justify your answer with
clear working.
Answer [3]
DllS 201 I Preliminary Sec 4 SAP Mathematics Paper 1(4016101) [Turn over
~ OHS 2011
8
Q M p
3cm
0
Answer (b)(i)
(b)(ii)
©OHS 2011
9
The diagram below is made up of a square, a regular hexagon and part of a regular n-sided
polygon. Given that the sides of the square, regular hexagon and the regular regular n-sided
polygon are of equal length and vertex A is joined to vertex C, find
(a) the value of y,
(b) the value of n,
AB A
(c) the value of - .
AC
(b) [2]
(c) [3]
DHS 2011 Preliminary Sec 4 SAP Mathematics Paper 1(4016101) (Turn over
C OHS 2011
10
13 (a) On the Venn Diagram shown below, shade the set (Av B ') n C.
Answer (a) ~
©OHS 2011
- 11
fn the diagram. A is a point on the y-axis and BC is a horit.ontal line. Gi\·cn that the coordinates of
B is (6. I) and the gradient of AB is - f. find
la) the length of AB.
(b) the coordinates of C gi\'en that the line AC is parallel to the line J = -:!x,
(c) the perpendicular distance from C to 'he line. IB.
B (6, l)
· f/S 2011 Preliminary Sec 4 SAP Matltemntics Paper 1(4016101) [Turn over
C OHS 2011
12
15 A car and a motorcycle travelled from Bugis to Kallang and the speed-time graph of their journe
is shown below. Both vehicles left Bugis together and took the same route.
motorcycle
Speed
(m/s)
20
0 2 4
Time
(seconds)
Find
(a) the initial acceleration of the car,
(b) the distance travelled by the motorcycle in the first 4 seconds,
(c) the time tat which the motorcycle overtook the car. (You may assume that t > 2.)
(b) ································ m
(c) .......................... second
Miss Fang found a bag of gold and she decided to bury it in a hole T, which is found within a
triangular field PQR as shown. It was given that Tis equidistant from PQ and QR and equidistant
from P and Q.
.t!nswer (a) and (b)
(a) By constructing suitable lines, locate and label the position of T [3]
(b) Mr Tan found the treasure and decided to bury it at a point S such that PQRS forms the
shape of a kite, where Sis outside the field. Mark out the point Son the diagram. (1]
DHS 2011 Preliminary Sec 4 SAP Mathematics Paper 1(4016101) [Turn over
0 OHS 2011
14
17 ABCD is a rectangle and Eis the midpoint of DC. The line BE intersects AC at G and the line AIJ
produced meets BE produced at F.
F
(a) Show that triangles GEC and OBA are similar.
Answer (a) In 6s GEC and GBA ................................................................. .
. . . . . . . . . . . . . . . . . . . . . . . . . . . . . . . . . . . . . . . . . . . . . . . . . . . . . . . . . . . . . . . . . . . . . . . . . . . . . . . . . . . . . . . ... [2J
(b) Name two triangles that are congruent.
Answer (b) ............... and ............ [11
(c) Find
Areaof 6GEC
(i)
Area of 6GBC '
Areaof !1FDE
(ii)
Area of A BCD '
Answer (c)(i)
DJJS 201 !Preliminary Sec 4 SAP Matltemalics Paper I (4016101)
7f
©OHS 2011
15
2
(a) (i) Express x 2 + 2x-6 in the form k(x+ a) +b.
(ii) Hence, sketch the graph of y = x 2 + 2x-6, indicating clearly the coordinates of the
x-intercept(s) and y-intercept, if any.
Answer (aii)
(3]
DHS 1011 Preliminary Sec 4 SAP Mathematics Paper 1(4016101) [Turn over
COHS 2011
16
(b) The point (I, I) is marked on each diagram in the answer space. On these diagrams, skctd
the graph of
(i) y = x 3 +1.
(ii) Y =l.y •
Answer (b)(i) y
• (L l)
[2]
(b)(ii) y
• (l. l)
[I J)
©OHS 2011
17
D
19
In the diagram above, OAB is a triangle. C is a point on AB such that AC : CB= 1 : 3. The side OB is
produced to point D such that OB : BD = 3 : 2. It is given that OA =a and OB= b. Express, as simply as
possible, in terms of a and/or b,
Answer (b)
................................................................................................................... [1]
Answer (d)
.................................................................................................................. [1]
l @DHS2011
: l&l\ · 2011YEAR4 SAP
MY~yna PRELIMINARY EXAMINATION
d eport n MATHEMATICS Answer Key
Paper 1
• \
-v7-1\
I
C 1../1-1
.- ..
\_;
1-1. -1)
-6
- 10
l (bi)
(bii)
Smaller pizza
33.7°
19 (ai)
(aii)
(aiii)
b -a
1.a
4
~b
+ lb
4
--3 3
5
12 (a) 120 (c) 1b-ila
3 17 I
(b) 12
(c) 0.518
(
13 (a)
~
16
p
18(bi) (bii)
Y y = x 3 +1
y
y=;
(1, 1)
I
2
4
(b) The cumulative frequency curve illustrates the marks obtained, out of l 00, by 400
DHS students in the recent Year 4 Mathematics Alternative Assessment.
Number of students
with this mark or
less 400 I I l I 1
360
I I
320
_I, I I II
I
I I
_l_l I
I ..
280 I I! I
240
200
160
120
80
40
20 40 "50 60 70 80 90
CDHS20 11
s
(ii) Given that 65% of the students passed the test, use the graph to find the passing
mark. (2)
Two candidates were selected from the 400 students at random. Calculate the
probability that
3 (a) The table below shows part of a utility bill received by Miss Fang in the month of Jun 2005.
Amount
Water Services by Public Utilities Board Usage Rate($) ($)
(iii) In the year 2012, the water consumption tax is reduced to 90% of the value in
2005 but G.S.T. is increased to 7%. Calculate the amount of G.S.T. that Miss
Fang has to pay. (You can assume Miss Fang still uses the same amount of
water and retains 2 fittings of sanitary appliances and the rest of the rates
remain unchanged.) [3]
A= { x: J; is an integer}
4 The diagram below shows a circle with centre 0 and radius OB. AC is a tangent to the
circle at point B. Given that OB= 7 m, AC= (12 + 2.x) m and OA = OC = (10- 2.x) m,
(iv) Solve the equation 3x2 - 52x + 15 = 0, giving both answers correct to four decimal
places. (3)
(v) Hence, find the length of AC, giving your answer correct to the nearest centimetres. [2]
- - --(12+2.x)m---
0
8
5 The diagram below shows the cross-section of a regular paper weight in the shape of a
circle of radius 8 cm. A regular octagon with side AB is inscribed in the circle and another
regular octagon with side IJ is inside the circle with OJ= OJ= 6 cm.
(iv) The portion of the paper weight with the segment AQB as its cross-section is filled
up gold dust water. Calculate the volume of gold dust water required if the paper
weight has a height of 10 cm. [2)
Q B
.....
-,
..
),olo ~
A
-,
. ___/
..
.
L
....
........
..
.,,i.; "~
~
.. ~
C :>HS 2011
9
6 In the diagram below, a circle with centre 0 is shown. Given that angle BCE =109° ,
angle CEF =110°. AB = AE and BE is parallel to CD, calculate, showing your reason(s)
clearly,
F
lU
7 To improve the situation of the Mass Rapid Transport (MRT), a team was commissioned
by the new Minister for Transport to do some research.
The following data was collected from the team studying the East-West Line:
During peak hours (i.e. from 7 to I 0 am and 5 to 8 pm) of every weekday, each carriage of
the MRT carries an average of 120 adult passengers and 40 children.
During off-peak hours of every weekday, each carriage carries an average of 56 adults and
56 children.
(i) Express the above information as a 2 x 2 matrix, C. [ 1]
On Saturdays and Sundays, each carriage carries an average of 144 adults and 46 children
during the peak hours (i.e. from 10 am to 1 pm and 4 to 7 pm).
Each carriage carries an average of 60 adults and 18 children during the off-peak hours on
Saturdays and Sundays.
(ii) Represent the above information as a 2 x 2 matrix D. [I]
8 The diagram below shows the maximum number of intersections obtained from 2, 3 and 4
lines respectively.
(i) Copy and complete the 4lh and 5th rows of the table. (4)
1 1 0 2= 1 +1
2 4 - 2{1) 4=1+3
1--
2
3 9 - 3(2) 7= 1 +6
3--
2
4 6 11=1+ 10
(ii) Hence, copy and complete the klh row of the table, leaving your a nsw er in term s of
k only. p:
DHS 2011Sec4 SAP Mathematics Preliminary Examinations Paper 2 (401612) (Turn over
<ODHS2011
9 In the diagram (not drawn to scale), ABCD is a rectangular piece of land on horizontal
ground where AC and BD intersect at a point 0. It is given that AD= 200 m, CD = 100 m
and C is due North and due East of D and B respectively.
(a) Calculate
(b) A vertical pole is erected at point 0 and a tent is set up so that ABCDP forms a
pyramid.
(i) Given that PD = 113 m, calculate the angle of depression of point D from the
top of the pole, P. [2]
(ii) Calculate, correct your answer to one decimal place, the volume of the tent,
ABCDP. [3]
A 200m
To test out a Physics' concept on Kinematics, Nicholas stood at the edge of the Sky Park,
which is at the top of Marina Bay Sands Hotel.
The height, S metres, of the paper clip above Sky Park at a time t seconds after it was
thrown is given by the formula
s = 3.6t-4.9t 2
The table below show some values of I and the corresponding values of S, correct to the
nearest whole number.
On your axes, plot the points given in the table and join them with a smooth curve. [
(i) the time when the paper clip was next at the starting position of being thrown
up, r
(ii) how far below the Sky Park Jerome was if he saw the paper clip 4 seconds
after Nicholas threw it vertically upwards. (
(d) (i) By drawing a tangent, find the gradient of the graph at ( 4.9, -100). [
(ii) Explain what your answer to ( d)(i) tells you about the motion of the paper clip
at t = 4.9. (
(e) Given that the paper clip stopped moving 6.67 seconds after being throv.TI vertically
upwards from the Sky Park, explain the significance of the value S = - 194 when
t =6.67.
--- Encl of Paper ----
DHS 201 I Sec 4 SAP Mathematics Preliminary Exammations Paper 2 (401612)
0 DHS201 l
Paper 2
2 6 8
I i 9 I 4
10 12
16
14 18
- -
3 5 7 11 13
15 17 19
7 (ii) 10
144 46) or ( 144 60)
( 60 18 46 18
- '"I,,-
0 -..i •1::::~ H -- , 1'"1"" "i
1 ·n
H f- rr-- -:'- ·1~ t '~ .·· -· •·rr
• I .... t· , ",...I·
t-.t- t:i..
(iii) 7104 2336) or (b)
( 3200 2528
7104 3200)
( 2336 2528
(iv) Total number ofMRT commuters
who commute via the East-West
Line in a week in categories of
age group and operating hours
(v) (6x 63 ° 180)=(120 180)
(vi) = (2163 840)
8 (i) 6, 25 , 10, 16
(ii) 2 k(k-1) (k+I)k
k , - 2- , l + - 2-
9 (a)(i) 112 m
(a)(ii) 063.4°
( a)(iii) 116.6°
(b)(i) 8.30
11 109341.5 m3
10 (a) 1
(c)(i) 0.7 to 0.8 s
( c)(ii) 60 to 68 (64 is the accurate
answer)
( d)(i) -44.42
(d)(ii) The paper clip is travelling with a
speed of 44.42 m per second in
the downwards direction. (i.e.
opposite to the initial direction of
going up)
(d)(iii) 194 mis the height of the Marina
Bay Sands building or ground
zero is 194 m below the S Park
I
Candidate Name: I Class: IIndex No:
2H 17 AUG 2011
Write your name, index number and class in the spaces provided at the top of this page
Write in dark blue or black pen.
You may use a pencil for any diagrams or graphs.
Do not use staples, paper clips, highlighters, glue or correction fluid.
This question paper consists of 16 printed pages including the cover page.
Compound Interest
Mensuration
Curved surface area of a cone= 7rrl .
Trigonometry
a b c
--=--=--
sinA sinB sinC
a 2 = b2 + c2 - 2bc cos A .
Statistics
Mean = i1.
Standard Deviation= Ifx-i
- --
IJ
(I__/!_
IJ
2
)
201 JI Preliminary Exam/ Elementary Mathematics/ Sec 4£5N Paper I Dunman Secondary School
c;
3
1. (a) 8.0lx25.0l
Use a calculator to evaluate
1.999
Write down all the figures shown on your calculator display.
2.
Consider the following numbers: ../65, -3.14, rr, o.4s, (-0.2)3, (-3)2 , ~-64, 3.!..
7
Write down the
(a) positive numbers,
3. 1 4 1 ·
x+-
So lve - - =- · the fiorm
, eavmg your answer m a±.Jb , w here a, b and c are integers.
·
2x+3 6 c .
Answer x= ....... .. ... . ... ..... ...... ....... ..... ........... ...... [2]
201 JI Preliminary Exam/ Elementary Mathematics/ Sec 4ESN Paper I Dunman Secondary School
4
4. (a)
Given that ( i J= 025, find the value of k.
2
5. Factorise fully 4 + 2ab - a2 - b .
Answer (2}
6. The scale of map P is 1 : p and the scale of map Q is 1 : q. If the same distance is represented
as 7.5 cm on map P and 11 .25 cm on map Q, calculate the ratio p: q.
Answer [2]
10111 Prelimmary Exam/ Elementary Mathematics/ Su 4E5N Paper I Dunman &condary School
98
5
7. Xin Ee is n years old, Yao Sen is 3 years younger than Xin Ee and Zhan Hong is the youngest
at 6 years old. Find an expression, in terms of n in its simplest form, for
(a) the (positive) difference in age between Xin Ee and Zhan Hong two years ago,
20111 Pn//m/nary Exam/ EkmentaryMathemallc.sl Sec 4EJN Paper I Dunman S.condary School
6
(a) FindLCDE.
Ans1.ier (a) LCDE= ..................... 0 [I]
(b) Given further that CD = DE, find
(i) LAED,
Answer (b)(i) LAED= .................. 0 (JJ
(ii) LABF.
Answer (b)(ii) LABF= ................... 0 ll I
20111 Prelimlllary &am/ Elementary Mathematics/ Ste 4E5N Paper I Dunman Secondary School
Candidate Name: Class: Index No:
11. Three unbiased dice are thrown. Find the probability that
(a) the three dice show different numbers,
12. It is given thats= {xis an integer: 50 ~ x ~ 100}, A= {x: 55 < x < 100} and
(b) AnB,
201 II Preliminary Exam/ Elementary Mathematics/ Sec 4E5N Paper I Dunman Secondary School
8
13. The force of attraction between two magnets is F newtons. This force is inversely proportionaJ
to the square of the distance, d cm, between the magnets.
(a) When the magnets are 6 cm apart, the force is 0.5 Newtons. Find the distance between
the magnets when the force is 0.72 Newtons.
(b) Hence state the least integer value of x which satisfies the inequality in part (a).
20111 Preliminary Exam/ Elementary Mathemat1cs/ Sec 4£5N Paper I Dunman Secondary School
102
9
15. Dunman Confectionery produces large, medium and small cakes. The table below shows the
inputs to produce each type of cak'!.
Labour (minutes) Flour (k~) Utility (units)
Large 45 0.6 5
Medium 15 0.3 3.5
Small 12 0.2 2
Labour costs $6 per hour, flour costs $2 per kg and utility costs 10 cents per unit.
3~5) ~ ).
(a) 0.75 0.6
Let A = 0.25 0.3 and B =( Given that C =AB, find C.
(
0.2 0.2 2 0.10
Answer (b)
[l]
16. The possible scores for each time a game is played are 1, 2, 5 or 10. The frequency table below
shows the scores for a certain number of games played.
1 2 5 10
Score
Frequency n+l n+3 n+4 n+3
(a) Find the value of n if the mode and mean scores are the same.
101 JI PrelimlflQl')I Exam/ Elementary Mothernotlc.JI Sec 4ESN Poper I Dunman Secondary School
10
Answer (a) [
Answer (b)
20/ JI Preliminary £:.corn/ Elementary Mathematics/ Sec 4£5N Paper I Dunman Secondary School
104
11
Answer (c)
Distance (m)
[2]
40 100 130
20111 Preliminary Exam/ Elementary Mathematics/ Sec 4E5N PaJHr I Dunman StcondlJry School
12
19. (a)
On the diagram below, sketch the graph of y =-2 . The point (I, l) is marked.
x
Answer (a)
y
Answer (b)(i)
y
(ii) Write down the equation of the line of symmetry of y =9-(x+ t)2.
Answer (b)(ii)
20111 Preliminary Exam/ Elementary MathematicJ/ Sec 4E5N Paper I Dunman Secondary Schoo/
10
13
20. The points A(-5, 5), B(l, -3) and C(4, -3) are shown in the diagram below.
y
A(-5, 5)
B(l,-3) C(4,-3)
20111 Preliminary Exam/ Elementary Mathematic.JI Sec 'E5N Paper I Dunman Secondary Schoo/
14
21. (a) The distance of the moon from the earth is 2.56x 10-3 Astronomical Units (AU). Given
that 1AU = 1.5 x l 0 11 m, find this distance in kilometres and in standard fonn.
22. On his birthday, Xin Tong put some money into an empty money box. Each week after that, he
puts a fixed amount of money into the box and does not take any money out. The following
table shows how much money is in the box after a certain period of time.
No. of weeks after the birthday 3 4 5 ... a . .. b . ..
Total amount of money($) 22 25 28 ... 58 ... c ...
201 JI Prtlimlnary Exam/ Elemen1ary Mathtma/lcsl Sec 4E5N Poper I Dunman &condary School
108
15
23. In the diagram below, QR= 7.5 cm, AB= 5 cm and QR II AB.
R
7.5 p
[2]
(b) Find
(i) Area of .6.ABC : Area of .6.RQC,
201 II Prelim111ary Exam/ Elementary Malhemallcsl &c 4E5N Paper 1 Dunman &condarySchool
16
24. (a) Draw a horizontal line segment AB 10 cm !ong in the answer space below. r11
(b) Construct the triangle ABC in vvhich LBAC = 50° and AC= 9 cm. [I]
(c) Construct the perpendicular bisector of AC. [I]
(d) Construct the bisector of L4CB. [I I
Answer (e) The point Mis equidistant from the points .................. and ................. .
and equidistant from the lines .................. and . . . . . .. . . . . . . . . . . . . . [I]
End of Paper
20111 Preliminary Exam/ Elementary.• Mathema11csl Sec 4E5N Poper I Dunman Secondary School
DUNMAN SECONDARY SCHOOL
PRELIMINARY EXAMINATION 2011
SECONDARY 4 EXPRESS I 5 NORMAL (ACADEMIC)
ELEMENTARY MATHEMATICS PAPER 1
Marking Scheme
•3 .z.s -z - 1.s ·l ·O S
l
0
D o. s
•
1 1. s z z.s )
•
•l
-z
-3
I *B2
-MDP
- of
-Area - - =DP
- ~
" "DP -_ 40 cm2
Areao f LV1 Al
Area of MBP BP
24 (a)
c
(b)
(c)
(d) ,.
MATHEMATICS 4016 I 2
Wnte your name, index number and class on all the work you hand in.
Write in dark blue or black pen on the writing papers provided.
You may use a soft pencil for any diagrams or graphs.
Do not use staples, paper clips, highlighters, glue or correction fluid.
The number of marks is given in brackets [ Jat the end of each question or part question.
The total number of marks for this paper is I 00.
This question paper consists of!! printed pages including the cover page
MrD.Wong
Mathematical Formulae
Compound Interest
Mensuration
Curved surface area of a cone = 1t rl .
2
Curved surface area of a sphere = 41t r
1 2
Volume of a cone= -7t r h
3
4 3
Volume of a sphere = -1t r
3
Area of triangle ABC= _!_be sin A
2
Arc length :::: re .where e is in radians .
1
Sector area= - r2e , where 0 is in radians.
2
Trigonometry
a b c
--=--=--
sinA sinB sinC
2
a 2 :::: b2 + c - 2bc cos A .
Statistics
"i.fx
Mean=
'L!
l. N
B
290m
c
A, B and C are three markers on a school field. Vanessa walks from point A on a bearing of 065°
to a point B which is 21 Om away. She then walks from point B on a bearing of 212° towards
(a) Calculate
(b) Vanessa erected a flagpole P that stands vertically at the marker B. Given that the angle
(ii) find the angle of elevation of the top of the flagpole P from the point Mon AB. [1]
(ii) Find the number of days it will require for 6 men to paint 4 bridges. [l]
20JJ/ Preliminary E.xoml Mathematics/ Ste 4EISN Poper 2 Dunman Secondary School
4
(d) Given that£ = {x: xis a positive integer such that 0~x~10},
P={x:x2 <30},
Q =(x: xis a prime number}.
(i) Draw a Venn diagram showing E , P and Q and place each of the members in [2]
(ii) [ 1]
Find n{PnQ).
(iii) [ 1)
Find n{PUQ').
3. Kenneth and Samuel visited a car showroom together and each decided to get a car of the same
(a) Kenneth paid a cash deposit of 20% and took a loan on the balance at a flat rate of 1.5% [3]
per annum. He paid the loan in equal monthly instalments over 5 years. Calculate his
monthly installment.
(b) Samuel was given a VIP discount of 0.5%. He then traded in his old car and paid the [2]
balance of $50,075 in cash. How much did he trade his old car for?
(c) How much more did Kenneth pay for the car than Samuel? Express your answer as a [2]
(d) After buying the car, Kenneth sent his car to a workshop in Malaysia for servicing. The
exchange rate between Singapore dollars and Malaysia ringgit was S$1 =RM $2.39.
(i) The total cost of servicing was RM $294, including a 5% government tax. [l]
How much did Kenneth pay in ringgit for the servicing?
(ii) Given that Kenneth changed S$200 initially, how much Singapore dollars did (2)
he have left?
5
4. Two taps A and Brun water at different speed. Tap A runs water atx litres per minute. Tap Bruns
water at a rate of 5 litres per minute faster than tap A. A rectangular tank with dimensions of 300
cm by 250 cm by 120 cm is to be filled with water. It takes 5 hours longer to fill the tank with
(b) Write down an expression, in terms of x, the time taken to fill the tank by using
(e) Hence find the time taken, in hours, to fill the rectangular tank if both taps A and B [2]
5.
A Q
The diagram shows two regular octagons that meet at points A and B.
(a) Find the size of each interior angle of the octagons. [1]
(b) Find LPAQ. (2]
(d) Hence or otherwise, find the area of the smaller octagon if the area of the bigger [2]
20111 Prd1mmary Exam/ Ma1hrmo1icsl Src 4EISN Paper 2 Dunman ~condary School
6
(a) C is the point (-8 , 5). The point D is such that DC= ( ~).
(i) Calculate I DC I. [1)
(b) In the diagram, F and Gare points on the lines AE and BE respectively. The lines BE
(i) Express and simplify each of the following vectors in terms of p and q.
(a) [1]
EB
(b) [1]
AC
(c) (l]
EG
(d) (1)
AG
20111 Pnliminary Exam/ Mothtmatksl Ste 4EJ5N Poper 2 Dunmon Stcondory School
G
(b) The load is lifted from D as the jib is rotated in a vertical plane about A. When the jib is in
the position AG, the load is lowered to the point Fon the ground, vertically below G. The
(ii) the angle through which the jib has rotated. (2)
minutes and 120 minutes. Choose a suitable scale for the vertical axis and draw a histogram
(b) In which interval does the median of the distribution lie? (I]
(c) Calculate an estimate of the mean time taken to complete the journey. [2]
(d) Two drivers are chosen at random. Expressing each answer as a fraction in its lowest terms,
(i) both took more than J10 minutes for the journey, (2)
(ii) one took 80 minutes or less for the journey and the other took more than 110 minutes. (2]
20lll Preliminary Exam/ Mathematics/ Sec 4E/5N Paper 2 Du11man Secanllary St'hool
8
9.
I
I
I
I
A---------E\J-------;
I ,'
C
I ,'
I ,,,"'"' r
!.-'
0
E D
Diagram I Diagram II
Diagram I shows a barn and Diagram II shows the cross - section of its end.
A farmer needs to order a new roof for his barn. The roof is represented by ABC, the arc of a
circle of radius r, centre 0. ABCDE is a rectangle.
The farmer measures AC, CD, BF and the length of the barn.
(a) Given that AC= 8m and BF= 2m,
(i) write down an expression, in terms of r, for the length of OF, (1)
(ii) show that the radius, r, of the circle is 5 metres. [2]
(b) Show that angle AOC is approximately 1.85 radians. (2)
(c) Given that the length of the barn is 12 metres, calculate the curved area of the roof [3]
(shaded in Diagram I).
(d) Given also that CD= 7 metres, calculate the volume of the barn. [4]
20111 Prtlimiltary Emmi MatMmDJicsl Ste 4F.15N Paptr 2 Dwnman Secondary School
122
2011 PRELIMINARY EXAMINATION
SECONDARY 4 EXPRESS I 5 NORMAL (ACADEMIC)
MATHEMATICS PAPER 2 Marking Scheme
·()
p
I 7
sets P and Q
3 5 Correct values in Bl
E set
~ -
6 8 9 10
(dii) n(PnQ)=4 Bl
(diii) n(PUQ') =9 Bl
~ )-(~)
(aii) 8 Ml
OD=( D = (-12, 2) Al
Al
8 (a)
L."I l I
Correct scale Bl
Accurate plotting of points Bl
Smooth curve + label axis Bl
6.5 m (±0.1) Al
It sto movin ./It landed on a int on the slo Al
Drawing of appropriate tangent at t = 4 Bl
Gradient= 1.13 (±0.1) Al
(dii) Al
(e) 9.72 -1 Al
Average speed =-- = 1.62 ms
., t-· i· t ITT1
.,.1 ~~
I
. ....... -rl+
...
·r
+
. I
- +
.. -iH
rq ;- -.. '
' !- ...t .. .
~i
.j.
....
. ~ +·
++.·
d
' r H·
:t r+ ·~ ·i+ .
-ti- . -
-1- 1-t+ j.. ,..;...·.;:
.--
I
. . 't""
I
i- ~
r-1:
. U:tt
. r
... #.l. ~· t: -r·
... .'t
'
~
j: -l . +n
.......
~.J
+ H.
. ~ I,. f.+. H·
'-1-
I
'"!- i.r-H
..
-
~
t '.i f ~i:
fJfi .
Lt.,.. -, rt
- • 't "'-!
. ,...
~
. l.:tr
'
ll I I
'
a:
rn·~ ~ ·id11 ! , , ~ * ·~~!!i "'- t ~wrr· trt ~ :i4:r t-rt- ·+
'..f
' I I
w • 4 I .... '• ...
PRELIMINARY EXAMINATION
Secondary 4
Write your name, class and index number on the question paper.
Write in dark blue or black ink on both sides of the paper.
You may use a soft pencil for any diagrams or graphs.
Do not use staples, paper clips, highlighters, glue or correction fluid.
173
Page 2 of 19 F"'
...Exam1iwr'1
US!e
Mathematical Formulae
Compound Interest
Total amount = Pr.~ + _r_)n
100
Mensuration
Curved surface area of a cone = 1trl
Trigonometry
a b c
--=--=--
sin A sin B sin C
a 2 =b 2 +c 2 -2bccos A
Statistics
Mean= L ft
'Lf
174
Methodist Girls' School Mathematics Paper I Sec 4 Preliminary Exam 2011
Page 3of19 For
l!:::tamtner ·s
Use
1 Express 2x-3 - --
3 . Ie 1ractlon
as a smg " . m . its . Iest fiorm.
. sunp
2x 2 +7x-15 x+S .
123456
2 Evaluate , giving your answer correct to 1 significant figure.
4
19.67
2
-( 2%)
.)
(I
~ ,
Ieavmg
. your answer m
. positive
'_ . .md.ices,
2
( 3c) x4c•
4 (i) Find the sum of all prime numbers less than 10.
(ii) In a shop, the ratio of the number of televisions sets to MP3 players is 8:3. If there
are 35 more television sets than MP3 players in the shop, calculate the number of
MP3 players in the shop.
(i) calculate the cost of US$ 3.3 trillion in Singapore dollars, leaving your answer in
standard form and to 3 significant figures.
(ii) Find the cost, in Singapore dollars, of an iPad 2 which is sold at US$499, giving
your answer correct to the nearest S$ l 0.
6 The capacity and height of the following two containers are the same. Wine is added at a
constant rate into each container until they are full. The total time to fill each container is
8 seconds. Sketch the graph for each of the containers.
(i)
Height (cm)
,,
cm
,,
,'
0 2 4 6 8
Time (sec)
[I)
Height (cm)
(ii)
2y
ycm
ycm
0 2 4 6 8
2xcm Time (sec)
[l]
1
Methodist Girls' School Mathematics Paper I Sec 4 Preliminary Exam 2011
Page 6of19 For
CXQmlMT
Ust
7 A train started its journey from Singapore at 15 26 and arrived at Gemas station,
Malaysia at 20 14 on the same day.
(i) What was the time taken for the journey from Singapore to Gemas station?
9 The table below shows the tax rates for resident individuals for year 2007 to 2011.
Chargeable Income Rate Tax (S)
First $20,000 0 0
Next $10,000 2 200
First $30,000 - 200
Next $10,000 3.50 350
First $40,000 - 550
Next $40,000 7 2 800
First $80,000 - 3 350
Next $40,000 11.5 4 600
First $120,000 - 7 950
Next $ 40,000 15 6 000
(ii) Mary paid an income tax of $855 in 201 1. What was her chargeable income correct
to the nearest dollar?
(ii) $ . . . . . . . . . . . . . . . . . . . . . . . . .. . . [2]
10 Given that a 2 -b2 =60 and a-b =6, find the value of
(i) a+ b,
(ii) Q2 +b2.
Answer (i) . . . . . . . . . . . . . . . . . . . . . . . . . . . . . .. [ 1]
13 The marks in a Mathematics test of two groups of students are presented in the following
back to back stem-and-leaf diagram.
(ii) Find the ratio of the number of students who scored above 50 marks in group A to
those of group B.
(ii) . . . .. . .. . . . . . . . .. . . . . . . .. .. . . . [ 1]
181
Methodist Girls' School Mathematics Paper l Sec 4 Preliminary Exam 20 J I
Page 10of19 For
ba,,,lncr~
Uu
14
A
c
In the figure, 0 is the centre of the circle. DOB is the diameter of the circle,
LCEB = 46° and LEAB = 132°. Stating your reasons clearly, calculate
(i) LBOC,
(ii) LDEC,
(iii) LEED.
15 {i) Dwing the Great Singapore Sale, the price of a Blue-Ray DVD player was
reduced from $240 to $212. If the player was sold at its original selling price,
the shopkeeper would make a profit of25%. Find the percentage profit when
it was sold at the reduced price.
(ii) The table shows the rate of interest offered by two different banks.
NameofBank Interest Rate
POSB 2% per annum compound interest
HSBC 4% per annum simple interest
Mr Tan deposited $1500 in POSB and $4500 in HSBC. Calculate the total
amount he would have in the banks from both the deposits at the end of two
years.
..,
(ii)$ .. ... . . .. ........... .. ...... [2]
1 f3
Methodist Girls' School Mathematics Paper I Sec 4 Preliminary Exam 2011
Page 12of19
Page 13of19 f
Lan
(..
17 The diagram below shows the graph of the equation y = (x + pX3-x) where pis a
constant. The graph meets the y-axis at A(0, 12) and the x-axis at B and C.
Y=(x+ pX3-x)
Find
(i) the coordinates of point B,
(ii) the maximum point ofthe~graph y = (x+ pX3-x).
(iii) A point ( w, 6) lies on the curve. What are the possible values of w?
Page 14of19
(i) Prove, stating your reasons clearly, that 6.A.BX is similar to MJCX.
c
(ii) Given that AB = 5 cm, CD = 9 cm and BX= 2 cm, calculate the length of CX.
(iii) Calculate, leaving your answer as fraction, the value of Area of 6.A.BX
Area of M>CX
Answer
... . .. .. . .. . . . . ... .. . .. . ... . ... . . ....... ......... .. ·:· .................. ................... ........ .... [2]
(i) --xz,
(ii) --
XM,
(iii) --
OM .
--+
(b) Express O y in terms of p, q and k.
v
2
(c) If k = - , what can you conclude about 0, Mand Y?
3
Answer (a)(i) -
xz =.. ........ ........ .. ..[I]
--+
0
(ii) XM =....... ............. [.l]
--+
(iii) OM = ... .......... ...... [1]
--
(b) OY =................ ..... .. [I)
20 The diagram shows a pyramid with a horizontal rectangular base PQRS and a vertex V
which is vertically above the comer S.
Given that PS= 10 cm, PQ = 24 cm, tan LVPS = 1.7, find
(i) the length of VS,
(ii) the numerical value of tan LSVQ,
(iii) the volume of the pyramid VPQRS.
p 24
(iii) .........................cm3 [ 1]
Methodist Girls' School Mathematics Paper I Sec 4 Preliminary Exam 20 l :
Page 17of19 For
Examiner·s
Use
21 When Elizabeth leaves for school in the morning, the probability that she takes her
umbrella to school is .!_ if it is a fine weather, and ~if it is a wet weather.
5 6
(a) Given that the probability that the weather of any particular day is fine is ~,
4
complete the tree diagram in the answer space.
Goes to school
with wnbrella
Fine
Weather Goes to school
without umbrella
Goes to school
with umbrella
Wet
Weather
Goes to school
( ) without umbrella
(2]
(b) Find, as a fraction in its simplest form, the probability that in a particular morning,
(i) the weather is fine and Elizabeth does not take her umbrella to school,
(ii) Elizabeth goes to school with her umbrella.
1 9 (bii) . . . . . . . . . . . . . . . . . . . . . . . . . . . . [2]
Methodist Girls' School Mathematics Paper I Sec 4 Preliminary Exam 2011
Page 18of19
0
22 The diagram shows the speed-time graph of a particle A during a period of 60 seconds.
The particle A starts from a speed of 10 mis and accelerates uniformly until it reaches a
maximum speed of v mfs. It then decelerates and reaches l 0 m/s at 36 seconds and .
continues to travel at this constant speed. The distance travelled by this particle A in th~
first 20 seconds is 280 m. Calculate
(a) the maximum speed, v, of the particle A,
(b) the retardation of the particle A during the motion.
Speed (m/s
25
Particle A
10
20 36 60 Time (s)
0
Another particle B starts from rest 20 seconds later, and accelerates uniformly until it
reaches a maximum speed of25 m/s at 36 seconds. It then continues at this maximum
0
speed.
(c) On the same axes above, draw the graph to represent the journey of particle B. [1]
Calculate
(i) the acceleration of particle B during the first 10 seconds of its journey,
(ii) the distance between particle A and particle B at 60 seconds.
23 Jannie wants to hide her school treasure in the field so that she can share her memories
with her daughter in future. In order to remember where she has hidden her treasure,
she comes up with a map. Please help Jannie draw her map in the space below.
(i) Given that the field ABCD has sides AB of 120 m and AD = 80 m which meet at
right angle. LADC = 110° and LABC = 55°, use a scale of 1cm to represent 10 m
to construct an accurate drawing of the field. [2]
(iv) Jannie hides her treasure in the field at the equidistant from A and D as well as
equidistant from the two sides BC and CD. Mark this point T and measure the
distance CT.
(v) There is a plan to set up in the field , a fitness comer of 120 m 2 • Calculate its area
on the map.
(ii) 80
3(i) I 9(i) 5, 650
--
12c 4
(ii) 44,357
x
(ii) -
3
4(i) 17 JO(i) JO
(ii) 21 (ii) 68
S(i) 12 11 (i) -2y2
4.05xl0
13(i) 57
(ii) 4:3
I • • '
0 14(i)
2 4 6 8 92
Height (cm) Time (sec) (ii)
44
z .. (iii)
42
6420.60
(ii)
16
16(1)
I I l I
• 2
0 2
I
4
I
6 8 (ii)
66- or 66.7
3
Time (sec)
J
No. Answers
0
No. Answers
20
l 7(i) ( -4 I 0 ) (i) 17
(iii) 1360
(iii) -3, 2
l 9(a)
6p-4q
(i)
3p- 2q
(ii)
4p
(iii)
(c) 2 - 3-
lf k = -, 0 Y =6p = -OM=> OYparallel to OM and O
3 2
is common. 0, Y and Mare collinear.
No. Answers
22(a) 18
(b) I
-
2
Speed (m/s)
(c) Particle B
25
Particle A
10
Time (s)
0 20 36 60
(i) I~
16
(ii) 56
23
CT= 6.2 cm
A B
(iv) 62
(v) 6
Class Index Number
PRELIMINARY EXAMINATION
Secondary 4
Write your class, index 9umber and name on all the work you hand tn .
Write in dark blue or bla'ck pen on both sides of the paper.
You may use a pencil for any diagrams or graphs.
Do not use staples, paper clips , highlighters, glue or correction fluid .
Answe r a ll questions.
If wo rking is needed for any question it must be shown with the answer.
Omission of essential working will result in loss of marks.
Calculators should be used where appropriate.
If the degree of accuracy is not specified rn the question, and if the answer is not exact,
give your answer to three significant figures. Give your answer in degrees to one decim al
place.
For n, use either your calculator value or 3 142. unless the question requires the answer
in terms of it .
At the end of the examination, fasten all your work secure ly together.
The number of marks is given in brackets [ ] at the end of each question or part
question.
The total number of marks for th ts paper s 100.
Marks
Compound 111reresr
11
Volume of a sphere
Trigonomefly
a h c
- - = -- = -
sin .·I sin H sin C
Statistics
>-1··
i\leun - - ·'
) .
-1
\ !.., l 'if
Page 2of10
3 2 '
. .fy x +3x +2x
1. (a) S imp11 . [2)
2x 2 -4x-6
(b) Express as a single fraction in its simplest form,
_ 5y-x
2 [2]
y+3x
2. A finn rents out small and large buses. The cost of rental of each bus is shown in the table
below.
(a) A small bus was rented out for 3 days and travelled 630 km during this time.
(b) A man who rented a large bus for -I days was charged $460.40. Calculate the total
(c) On one occasion, a small bus and a large bus were each rented out for 5 days. Each
travelled the same distance, which is greater than 1000 km. The total charge for the
large bus was$ I 39 .27 more than the total charge for the small bus. Calculate the
distance that the large bus travelled. [3)
(d) On another occasion, a man rented a small bus for one day and travelled 178 km. He
spent a total of $124.50 on the i·ental charge and petrol. Given that a litre of petrol
costs $I .96, calculate the; petrol consumption of the small bus, giving you r answer in
kilometres rcr litre. [3]
3. Carrie bought some wallets for $892.50. There were 12 wallets in poor condition. The cost
price of each wallet was $x. She sold all rhe wallets in good condition. Each was sold at $7
more than the cost price for her0 to make a profit of $253.
4. The figure shows the outline of a badge made up of three sectors with the same centre, 0.
AOB is a straight line. AOBCF forms a semicircle. The arcs AF, EPD and BC are each of
length 1.5 cm.
\ I
\ I
\ I
I
' \ I
\ I
\ I
\ I
\I
A 0
(iii) thr ru11~1i11ing area or the hm.lge with the segment EPD rcmo\'ed. [41
5. A large area is to be paved with blocks each one metre square. On Day 1, three
blocks are placed in a line, as shown in the diagram. Each followi ng day the paved
region is enlarged by adding blocks to surround the previous day's region, as shown
below
The perimeter and area of the total region CO\ ered by the blocks, after the completion
of each day's work arc calculated as in tht.: table below.
By considering the paved regions and the patterns developing in the table, answer the
questions below.
(ii) Write an express10!). in tenns of n. in its simplest form, for the perimeter
after Day n. [l)
(iii) Give a reason why I 002 cannnt be the perimeter after a certain day. [ lJ
(ii) 2
Write Jown an expression, in the l(inn an -b, for the area after Day n. ll J
0
(c) (i) Find an expression, in terms or 11. in its simplest form, for the number of
blocks added after Day n. [2]
(ii) Explain clearly "hY it is po:-;..;ihk IO add 2012 boxes in a certain day. [lj
M\!lhod isl <I ills' School Mathcm..t 1cs 1'.ipc1 2 Sec 4 Prclim Exam 2011
199
Page S of 10
6. P, Q, R and Sare four points of a field on level ground with R due east of Q.
27.1 Ill
(a) Calculate
(b) A girl, standing al a point inside the lii..:l<.l PQRS, spotted a kite on lop of a vertical
tree al R. I he angle of elevation ol the kite was 32°. She mo\ c<l forward a distance
of I 0 mctn.:s ll)war<.ls Rand found the angle of elevation of the kite to be 48°. Find
the height 111 thr tree. [2J
Methodist Girls' School Mathemat ics Paper 2 Sec 4 Prelim Exam 201 1
Page 6of 10
(b) Given that £ = {x: xis an integer, 2~x~15}, A= {x: xis a factor of 12}
and B={x:xisamultipleof3}
The follo,\ing table 1s used in calculating the cost of manufacturing each toy.
Labour costs $6.50 per hour, wood costs $1 per block and paint costs $1.20 per tin.
(ii) Represent this information in a 3 by I matrix. named B. [l)
(iii) Evaluat-.· C = AB [l J
F:.. p!,1111 what the clements in matrix C represent. [ l]
(iv) Givu1 J) = ( 150 225 ). evaluate DC and explain the significance of your
. [21
After it is baked. the cookie becomes a cylinder of radius 3 cm and height 0.6 cm.
(ii) Express this volume of air as a percentage of the volume of the mixture. [1)
Diagram J Diagram II
Three of the cookil.:s arc sh0\\11 in Diagram II.
0 is the centre or 1he hexagon in Diagram l :ind of the middle cookie in Diagram II.
(iii) 13) w11 ~ id<.:ring similar 1riangks. or othcrn ise. calculat<.: Ilic length of a side or
[2J
Methodist Gtrls' School M;1thcmaltl'S Paper 2 Sec --l Prelim Exam 20 I I 202
Page 8of10
(b) Using a scale of 2 cm to represent 1 unit, draw a horizontal x-axis for -1.5 s x s 3.5.
(c) Use your graph to write down the range of values of x for which the gradient is
positive. [1]
(d) Use your graph to find the value of x for which ~x 2 (3-x) =3.
2 (1)
(e) By drawing a tangent, find the gradient of the curve at x = 2.5 . [2]
(f) (i) On the same axes, draw the graph of the straight line x + y = 1. [I]
(ii) Hence write down the three possible solutions to the equation
1 2
- x (3 - x) = 1 - x. [I)
2
11ethodist Girls' School Mathe ma I ic.:s Paper 2 Sec 4 Prelim Exam 201 I
a "5" 7 V& JV
10. 70 rurmers took part in a cross-country race. At various times after the start, one of the
officials recorded the number ofrurmers who had finished the race. The official's results
are given in the cumulative frequency curve below.
10 20 30 40 50 60
Time (minutes)
(b) Calculate
0
(i) an estimate of the mean time required to run the race, 11]
(c) Another group of 70 runners have the same median but a sm a ller
standard deviation. Describe how the cumulative frequency will differ
[ 1]
Two balls arc.: drawn at random. one after the other, from the bag without replacement.
(i) Draw the possibility diagram to show the outcomes of the draw. ( 1)
(b}
( 89 ~\".SO -1.2 }x r 7)-89'2.50 = 253
(c) (1) .1=(65 43 3)3
(c) (ii) 6.50]
x= l2.75 or t =-'I0-5 B= I
6 (
1.20
{d) 58 (iii)
1--~~1--~~~~~~~~~v~~J
4(a) AO = OB (radius of circle)
(:~:~~)
OF = QC (radius of circle) The cost to manufacture 1 toy car= $45.60
I
I
The cost to manufacture 1 doll = $40. l 0
. . (equal nrcs subtc11
c.AOJ· = L.JJ( J( 1 (iv) l15862.50)
1,yqual Ls
. !::u I FO = IBCO (SAS) Total cosl of manufacturing 150 toy cars
an<l 225 dolls is$ 15862.50 .
or AF· tu· (t:qt1al arcs have
equal chord-;)
(b}(i) :. &IFO = WCO (SSS)
(ii} 0 .75
(iii) 8.5 cm
1
3 .24 cm
206
No. Answers No. Answers
8.(a) 1.53 cm I lO(a) (i) 43 minutes
fV
(b)(i) 1.96 cm (ii) 9.5 minutes
0 (1 , 2) (l , 3) (I , 4) (I, 5)
(ti) (a) 3
10
(b) I
-
5
(c) I
-
10
(d)
-109
9(a) 2
""""'"' :)\•
••••• •• · '·••-••• •• ·••••y.._..,, '''"''' ••••• •• • · •·•·••••·•• · ••·•• ••• ••••••••••o• •· •··••••••• · •••••••••••••
(b)
L:
r::(. ·· ::·:: :·i:.:·: ,: :'::.: c~i · c· :i::: : ::· t:· -~·· .
(d) x = -1.2±0 05
4016/1 MATHEMATICS
PAPER 1
Marks I 80
This Question Paper con sists of .1Q... print ed pag es, includ ing this p age.
127
3
For for
!xamtncr's
Answer all the questions. Exomincr"
Use Uot
I -0.00678 . .
1 (a) Calcu Iate ~ , correct your answer to I significant figure.
0.45x _.E_
1000
1
(b) Express 5- as a percentage.
3
2 Mr Lim's dinner bill in ABC restaurant was $170.665, before rorn1ding off to the nearest
cent. The bill includes I 0% service charge and 7% goods and services tax (GST).
Calculate
.:,~~er's 3 Write down an expression, in tenns of n, for the nrh tenn of the sequence I
Exa1
Use 1
(a) 2, 4, 6, 8, 10 ..... .
(b) [1]
4 A helium-neon laser emits light of wavelength, A.= 6.328 x io-s cm for a certain
frequency, /(Hz).
(b) Given that the wavelength, :>.. (in cm) of light is inversely proportional to its
frequency, calculate the new wavelength of light, in nm, emitted by the
laser when its frequency is now increased by 250%.
(a) Find n.
(b) Calculate the area on the map, in cm2, which represents an actual
area of 3 km2 •
6 At noon on a particular day, the temperature at the bottom of a hill was l o·c
and the temperature at the top of the hill was -12'C.
For for
::Xammcr's 7 (a) Factorise 1- a 2 + 2ab- b 2 • Examine
Use Us•
(b) Given that - 6 $ x < 4_!._ and -1_!._$y$4, find the largest possible integer
2 3
value of (x + y)(x - y).
(b) [l]
8 Solve
2
(a) (3m + 3) = 9, where m < 0.
(b) [2]
TKGS Preliminary Exam 2011 4016101 Mathematics Paper
7
For
Ex~~~•r'$ 9 One interior angle of a n-sided polygon is 84°. Ex.ami"'
Use Use
The rest of the interior angles of the polygon are each equal to 136°.
(b) Find n.
(c) Given that (n - a) xl 80° is the sum of its interior angles, find the value of a.
(a) [l]
(b) (1)
(c) [1]
10 (a) Simplify( 2x
3x
~ ]_,, leaving your auswer as a positive index.
(b) [2)
Number of siblings 0 t 2 3 4 5
Frequency 1 3 x 6 4 5
(a) If there are 35 students in the class, calculate the mean number of siblings that
the students have.
(b) If the mode of the above distribution is 2, state the smallest possible value of x .
(c) Given that the median is 3, write down the largest possible value of x .
(b) [l]
(c) [t]
(a) Find the value of m and of n given that 63 is a factor of n and not of m, m > n.
(b) Hence, find the smallest integer k, such that mk is a perfect cube.
Answer (a) m = ................ n = ... ... ... ... ... .... (2]
(b) k = ... ... ... ... ... ... ... ... ... ... ... ... ... .... [1]
Answer
(a) [I]
(b)(i) [1]
(b)(ii} [2]
For F'
1m1ner·s Exwr
Use 4 The diagram shows a triangle ABC, with AB parallel to the x-axis. u
A B
Given that A is ( - 2, 2), C is (7, -I 0) and the equation of the line BC is y =-2x + 4;
(a) find the gradient of AC.
(c) A point Dis added to the diagram to form a trapezium A BCD of area 60 units2 .
Find.
(i) the x-coordinate of D and
(ii) the equation of the line that passes through D and parallel to AC.
(b) [I]
(c)(i) (I]
(c)(U) [2]
TKGS Preliminary Exam 201 I 4016101 Mathematics Paper
12
3x-2 6-9x
(b) Simplify - - + 2
.
2x-6 x -4x+3
(b) (2]
For
Ex•mincr's 16 In a class of 40 international students, 23 students speak English, 8 students speak
Use German and 12 students speak French. 1t is given that
(a) Based on the Venn Diagram below and given that n((Fu E) 1
F
G
0
(b) Express in set language the following statement:
GcE
(b) [I]
(a) [l]
y
(b) Sketch the graph of 18(a) on
the grid provided.
Indicate clearly where it cuts the
axes.
[2]
seconds.
Speed (mis)
w
0 10 15 30 Time(seconds)
(a) Given that the distance the particle covered for the last 15 seconds is
75 m, find the value of w.
(c) On the axis below, sketch the distance-time graph of the particle's movement.
(2]
Distance
(m)
0 15 30 Time(seconds)
( d) Calculate the average speed, in km/h , of the panic le, for the first 30 seconds.
Answer
(a) [l]
(b) (1]
(d) [I]
For 22 (a) At 'QuickPrint' shop, the cost, C, in cents, of printing postcards consists of
miner'• E><m
Use a fixed charge of $1.00 and a charge of 6 cents per card. 1
(i) Write an equation to determine the cost, in cents, for printing n cards.
(ii) Draw and label the graph of the above equation on the grid below. [1)
(b) At 'SuperPrint' shop, there is no fixed charge and it costs 10 cents to print a
card.
(i) On the same grid below, draw another graph to represent the above
information. [ 1]
(ii) State the range of values of n for which it will be cheaper to print cards
at 'Quic.k.Print'shop.
c.
cents
400
300
200
100
n
0 10 20 30 40
Answer
(ai) (1)
(a)(iii) .. .. . . ... ...... ...... .. .. . . . . ....... .. .... ... .. . ... . . .. ......... .. ... ......... ........... ... [l)
(b)(ii) [I]
Fo
for 23 The diagram shows the cumulative frequency of the marks scored by 200 students in Ex.amil
E··~rnmer's the exams. Us-
Use
~
~ 160 -l-'-~·-------1-~~+--~-+-~-...~~--~-1-~--;-~~1-"----t
e
><
-+
••• i
.. . -
l...
0
i + ~
t
I
I
0 20 40 60 80 100 marks
(a) Complete the grouped frequency table based on the graph above. (I]
Number of students 6
Answer
(b)(i) [I]
(b)(ii} [l]
3
Map area= - X 25 .. 18.75 cm 2
4
6a HH-12) 22°C
6b 440
Height~ - x(l0-(-5)]
22
= 300 m
?a 2
l -a +2ab-b2
= 1-{a2 -2ab+b 2 )
= l-(a-b) 2
= (l + a-b x1 - a+ b)
7b (x+y)(x-y)
= x2 _ y2
= (-6)1 -(0)2
=36
Sa (3m+3) 2 =9
3m = 3 - 3 or 3m = -3 - 3
m - 0 (rej) or m=-2
:.m=-2
9a 96°
9b 96° +(n-1X1so 0 -136°)= 360°
n-7
9c a-2
IOa 9 l
- xl
9 3
-x 2 = -x
4 4
IOb 33(2>'-t) =32+.r
Comparing powers.
6x-3 • 2+x
x=I
Ila x= 16
3 + 2(16)+ 3(6)+ 16+ 25
:. Mean=
25
- 2.69 (to 3 s.O
lib x- 7
I le x JO
l2a m=2 3 X3Xll 2
:. m = 2904
n=2 2 x3 2 x7xll
:.n 2772
12b For mk to be a perfect cube,
m=2 3 x3x11 2
:.k =3 2 x ll
k 99
13a
13bi
P(2 balls taken arc green)= 83 X 72
3
=-
28
13bii 3 5
"Y(afleast rball 1s yellow)= 8X 7 X -:.l +58X 47
25
=-
28
OR
P(at least I ball is yellow)
=I - P( none of the balls are yellow)
= t-(~x~)
25
=-
28
14ci I
60=-(3+/)xl0./=9
2
x-coordinate is 0.
14cii -4
y = - x + c. sub (0. -/ 0)
3
-4
15a
y=
3
x-10 or3y+4x+30=0
I .
(-1, 0)
I ' . .'' I /1
(3,0)
' I
I
'
\.\ J I
'
''
I
I I
I
19a LAED = LBEC (vertically opposite Ls}
LADE = LBCE (Ls of the rnme segment)
~
I
I
'
(I, -4)
1
LEAD= LEBC (Ls of the same segment)
2 The table below shows the petrol prices ($) per litre for a particular grade of petrol
o ffieredb1y 3 petro l stat10ns.
Station A B c
Price per litre 2.040 1.938 2.035
($/litre)
Peter purchased $50, $80 and $60 worth of petrol from station A, B and C respectively.
1
2.040
1
(c) Evaluate (50 80 60 - - and describe what the matrix multiplication represents.
1.938
1
2.035
[2]
(d) Station A gives a 14% discount on its petrol price when paid with a bank's credit
card. Calculate the amount of petrol, in litres that can be purchased now with $50. [2]
3 A soda drink manufacturing company packs 8 cylindrical cans of soda in a crate as shown in
the diagram. Each cylinder has a radius of 6 cm and a height of 15 cm.
(a) Calculate
(i) the dimension of the crate required to contain the 8 cylindrical cans, (1)
(ii) the amount of empty space in the crate that is not occupied by the cylindrical [2]
cans.
The company decides to manufacture the soda in the form of smaller similar cylindrical cans
with radius x cm and height y cm instead. The volume of one such cylinder is I 000 cm 3 .
(b) Find
(i) the total volume, in m', for 2900 such cylindrical cans, [I]
(ii) x, [2]
(iii) y. [2]
6
Line I l+l--=- I
2
3
lx2
Line 2 l+---=--
I
2
l
3
7
2x3
Line 3 l+---=--
1
3
I
4
13
3x4
(ii) Express the n'h line of the sequence in a similar pattern. (1]
In a game, a button is placed on triangle X (starting point) and the players throw the
die twice and add the two numbers obtained.
(i) Draw a possibility diagram to show all the possible sums of throwing the die
twice. [2)
If the sum of the two numbers is an even number, the button is moved two triangles
in an anti-clockwise manner.
If the sum of the two numbers is an odd number, the button is moved one triangle in
a clockwise manner.
(ii) Find the probability that the sum of the two numbers is an even number. [1]
(iii) Find, as a fraction in its simplest form. the probability that the button finishes
at Q. [I]
15
TKGS Secondary 4 Preliminary Examination 2011 4016102 Mathematics
[Turn Over
8
(ii) .
H ence, sol ve th e mequa 1·1ttes
. - 1$ -x - -
3--x <2.
5 7
Represent your solution clearly on a number line. [3]
(i) Express y = 2(x 2 -2x + 4.5) in the form y = 2(x-h)2 + k where h and k are [2]
integers.
(ii) Write down the value of x for which the minimum value of y will occur. [1]
6 (a) In June 2011, the price of a car was $56 000. Over the next two months, the price
increased by 25% and then decreased by 15%. Mr Tan, a potential car owner, said
that the overall increase in the price of the car was l 0%. Do you agree with Mr [ 2]
Tan? Justify your answer.
(b) Mr Goh bought a sports car from a car dealer. He made a down payment of 40% of
the selling price and paid the remaining amount by taking up a monthly instalment
payment scheme jointly offered by Bank A and Bank B:
(ii) calculate his monthly instalment payable to Bank A and Bank B [4]
respectively.
After 3 years, Mr Goh sold his sports car to a Swiss friend at a loss of 15% of the
price he paid for it. The price he paid for includes the interests paid to the banks.
(iv) Given that he received 56 900 CHF (Swiss Franc), calculate the rate of [I]
exchange, in Swiss Franc (CHF) to one Singapore dollar (S$), giving your
answer correct to the nearest cent.
10
7 ABCD is a trapezium. AB= 18 cm, DC= 8 cm, AD= BC and the circle touches all the
sides of the trapezium.
A 18 cm B
(c) XW and XY are two chords of the circle as shown below. XY = 7.2 cm.
WY is the height of the trapezium.
A
Find
(c) The angle of elevation from B to the top of a tower at Sis 53°.
Find the height of the tower. [2]
(d) A boat sails directly from J to B. It moves at an average speed of 43.2 km/h.
Calculate
(ii) the time taken for it to reach B, correct to the nearest minutes. [2]
12
9 In a factory, a frustum-shaped iron pail, of height 9 cm and radii 1 cm and 4 cm, was made by
cutting out a right circular cone from a larger right circular cone.
Diagram I shows the frustum-shaped iron pail.
Diagram I
(a) Find
(i) the height of the cone that was cut off, [2]
(ii) the volume of the pail, leaving your answer in terms of 1t. [2]
(b) The pail is filled with waste liquid to a depth of 6 cm and spherical ball bearings of
diameter 4 mm are dropped into the pail.
(i) Show that the amount of waste liquid in the pail is 26;r crn3 . [2]
(ii) Hence, determine the least number of ball bearings needed for the waste liquid
to overflow. [2]
(c) Another way of making the iron pail is by joining the edges AB to DC of an iron
sheet, as shown in Diagram II.
The table below gives some values of x and the corresponding values of y, where
2400
y= 10+ -
. x
(b) Using a scale of 2 cm to 200 units, draw a horizontal x-axis for 0s;xs;1200 .
Using a scale of 2 cm to 5 units, draw a vertical y-axis for 0 s; y s; 40.
On your axes, plot the points given in the table and join them with a smooth curve. [3]
(d) (i) On the same axes, draw the graph of y =25 - - x for 0 $ x s; 1200 . [2]
60
(ii) The values of x where the two graphs intersect are the solutions of the
equation Ax 2 - 900x + B = 0. Find the value of A and of B. [2]
End of Paper 2
l (a) 120
-
x
(b) 232
--
2x+8
(c) ~=120 _2.75
2x+8 x
232x = (2x + 8)(120-2.75x)
232x = 240x - 5.5x 2 + 960- 22x
5.5x2 + I4x-960 = 0
l lx 2 + 28x- 1920 = 0
(d) -28 ± ~28 2 -4(11)(-1920)
x=
2(11)
x = 12 or -14.5(3s/)
(e) 232
=$7.25
2(12) + 8
2 (a)
(2MO]
1.938
2.035
(b)
(193.8
2M]
203.5
The elements represent the amount payable in each
station for 100 litres of petrol.
(c) (95.3)
The matrix represents the total amount of petrol in
litres pu:chased by Peter from the 3 stations.
(d) = 28.5 litres
3 (ai) 48 cmx24 cmx15cm
(aii) Amount of empty space
2
= 48x24x15-8(1t)(6) (15)
= 3710 cmJ,(to 3 s.f)
(bi) 2.9 mJ
(bii)
x= r,! = 5.031 = 5.03 (to 3 s.f)
(biii) 5 031
y=.::x15= · xt5
6 6
y =12.57698 ..
y =12.6
+ 1 2 3 5 7 11
l 2 3 4 6 8 12
2 3 4 5 7 9 13
3 4 5 6 8 1( 14
5 6 7 8 10 1~ 16
7 8 9 10 12 ]£ 18
11 12 13 14 16 H 22
.
(bii)
P(sum 1s even) = -26 = -13
36 18
(biii) 65
-
162
biv) P(at R) = 0
5 (ai) x 3-x
----
5 7
_ 7x-5(3-x)
35
12x-15
=
35
(aii) x 3-x
-1$----<2
5 7
.
-35$12x-15 and l 2x -15 < 70
2 1
x~-l- x<?-
3 12
2 l
:.-l-$x<7-
3 12
(bi) 2
y = 2[(x-1) -1+4.5)
= 2(x-1)2 + 7
(bii) The minimum value occurs when x = 1.
(biii)
y-7 =(x-1)2
2
4016102 Mathematics
162
TKGS Secondary 4 Preliminary Examination 201 I
[Tum Over
16
x=±~Y~ 7 +1
6 (a) Disagree with Mr Tan.
Or (1.25 x 0.85-1) x 100% = 6.25%
(bi) $31374
(bii) $949.94 (cor to 2 dp)
$936.49 (to 2 d p)
(biii) Selling price =
0.85 [41832+36(949.935 + 936.4947)]
= $93281.95
(biv) 0.61 CHF (nearest cents)
. LXWY =7-
sm ·2
12
LXWY = 0.6435 rad
(cii) Let the centre be 0.
LYOX = 0.6435x2 = 1.287 rad (Lat centre= 2 Lat circumference)
length of arc xzy = 6 x 1.287 = 7. 72 cm
(ciii) 16.1 cmi ( 3 s.f)
8 (a) - 1 1255
LSJB = cos ( - -) = 16.95° = 17 .0° ( I dp) or 0.296 rad.
1312
(b) BearingofB fromJ= 111.40+ 16.95= 128.4° ( 1 dp)
(c) h
tan 53°= - -
2600
Height= 2600 tan 53°
= 3450 m (3 sf)
(di) 1170 mz(to 3 sf)
(dii) 9 min ( nearest min)
9 (ai) Let the height of the cone that was cut off be x cm.
l x
-= - -
4 x+9
:. x= 3
Height of the cone that was cut off is 3 cm.
(bii)
Least number of ball bearings needed= ~ 16 ·254
-Ji(0.4f
3
= 433.59 ::::434 balls
10 (a) x = 13.4
(ci) 12.0
(cii) - 0.06 accept = 0.0587 [ ± 0.005]
(dii) x 2 -900x+14400 = 0
A = 1, B = 14400
Class Register Number
Name I l
4016/01 11/t1P2/EM/1
MATHEMATICS PAPER 1
VICTORIA SCHOOL
Write your name, class and register number on all the work you hand in.
Write in dark blue or black pen.
You may use a pencil for any diagrams or graphs.
Do not use paper clips, highlighters, glue or correction fluid.
At the end of the examination, fasten all your work 5,ecurely together.
The number of marks is given in brackets [ ] at the end of each question or part
question.
The total number of marks for this paper is 80.
Compound interest
Mensuration
Curved surface area of a cone = nrl
Trigonometry
a b c
--=--=--
sin A sinB sinC
a2 = b2 + c 2 - 2bccosA
Statistics
Mean = 'f.Jx.
If
4
3
311003 - 26 x 311000 +I
1 Evaluate
311001 + 3
2 Light travels 1 metre in 3.3 nanoseconds. How far will it travel in 11.55 picoseconds?
Express your ans~er in standard form.
Answer· ..........................:-................ (2 J
4 (a) Joseph sets off from his home to the Botanic Gardens in a taxi at 16 31 . Due to a
traffic jam, he did not manage to reach Botanic Gardens until 18 09. Calculate the
time taken, in hours, for Joseph to reach the Botanic Gardens. Give your answer in
exact form.
(b) Find the distance, in kilometres, between his home and the Botanic Gardens if the
taxi Joseph took. travelled at an average speed of 40 km/h.
x2 + y2
(b) smallest possible value of _ __:;__
y
(b) Find the smallest positive integer /1 for which 480n is a muluple of 576.
8 Study the following sequence. The integers from row I to 5 are consecutive even numbers
6
5
9 The temperature at the top of a mountain, of height 4000 m is -q °C. The temperature at
sea level is p °C where p and q are positive integers. Assuming that the temperature
changes uniformly with height, write down an expression, in tenns ofp and of q for the
(b) Find the equation of the line passing through B which is parallel to the line
2x=44-y.
6
6
11 The diagram shows the major sector of a circle,
centre B and radius 15 cm. The reflex angle
ABC = 216°. A cone is formed by joining AB and
BC together.
Calculate the 15 c
(a) radius of the cone,
A
(a) Ar1C,
(b) (AvC)'nB.
Calculate the
Calculate z R
(a) "
HPZ,
"
Answer (a) HPZ = ....................... [2]
,,
(b) PSR.
"
Answer (b) PSR = ........................ [I)
15 " = 2.4.
B lies on AC such that BC= CD. Angle ADC= 90°, BC = 5 cm and tan DCB
D
Find the exact value of
(a) AD,
A B 5 c
(b) " .
cos DCB
2a ~ )
2 3 )-2 + ( Sb-lcs
2a b c -S . . .. . .
(b) Simplify ( , leaving your answer m pos1t1ve md1ces.
5
17 ". survey was done to find the average number of hours each student in a class spcn
surfing on Facebook per day. The results are sho,.,n in the table below.
- um_b_er_o_r_h_o_u_rs_ ____.._'__.__2___.._3__._
N
'-Number of children 6 8 x
49_ I. 45 I. IJ
n3
(a) If the average number of hours each student spent surfing on Facebook per dn\ is
3.15 hours. find x
(c) If the median number of hours each student spent on surfing Face book is 4. fin<l the
possible values oft.
10
9
18 Mrs Tan will either cook food or bake a cake for her family during the weekend. The
probability that she cooks food is i.
5
If she cooks food for her family, the probability that
she will overcook her food is ~ . If she bakes a cake, the probability that she will bum the
8
. I
Cake I S - .
IO
Answer(a)
Cooks food
Wi 11 burn cake
Bakes cake
(b) Find the probability that she will not bum the cake or overcook the food during the
weekend.
(b) A fruit seller bought a box of 142 apples for $35.50. lfhc sells each apple for $0.44.
what is the least number of apples that he must sell in order lo make a pro fit of not
less than $12?
0 x
[2]
!3
- .L
11
22 The table below shows the number of cupcakes and muffins sold by two shops, shop A
and shopB.
Shop A ShopB
Cupcakes 150 139
Muffins 147 143
It costs $0.10 and $0.15 to bake a cupcake and a muffin respectively. Each cupcake was
then sold for $0.75 while each muffm was sold for $0.85.
(a) Write down a 2 x 2 matrix, Q, to represent the number of pastries sold by both shops.
Answer (c)............................................................................................................................ .
D c E
.. ..................................................................................................................... [2]
(b) Find
EY
(i)
EA '
0 r B 12 w
2
Answer (d) .............................. cm [2]
End of Paper
Tl11s document 1s 111tended for mternal circulation m l'tctoria School only No part of 1/11s document may be reproduced, stored in a retrieval
system or transmuted m an) form or l>y any mi!ans, electromc. mechamcul µhotocopymg or othern 1se, wtthout the pri11r perm1ss111n of the
Victoria School Jnrerna/ Exams Com111 111ee
. .
l
0 '
·6
·IS 1"\
I
I \
9(a) p+q oc 2 l(a) newy=3
9(b) 3 2 2l(b) 20 days
-p--q
5 5
lO(a) a=2 22(a)
(150 147)
Q = 139 143
lO(b) y=-2x+9 22(b)
(R-P) = (200.40)
Q 190.45
1 l(a) 9cm 22(c) It represents the profit earned by
Shop A and B respectively from
selling c upcakes and muffins.
l l(b) 12cm 23(b)(i) 2
-
3
12(a) {2,5} 23(b)(ii) I
-
I 2
12(b) (A u C)' n B = {9, 15 ( 23(b)(iii)
I 3.
5
13(a) 720 24(a) Scm
13(b) 60' 24(b) 7.5 cm
13(c) 240 24(c) 18.8 cm
14(a) " 24(d) 4.43 cmi
HPZ= 100
14(b)
-
I
~
PSR =44"
4016/02 11 /4P2/EM/2
MATHEMATICS PAPER 2
Friday 19 August 2011 2 hours 30 minutes
'IJC:~sc:HOOL. ~iere>A.. tcHool. VCn::lftti'SQ-OOt ~roiltfAICHOC:I(. Y'C:'°"1.AS'GHOOt. *'~ SO'fOOl VJICrQMt SCHOO(_ VK:~IC>IOOC. WC'°"'1tSOIOOl W:::7WUASc:HOOL WC:IOIMICHOot. YJC~SCHO«
W!CfOM& SCHOOi. VICM>ll:liot SOIOOC. \'JC.roRJA .SCHOOt 'l"CTOI™ KHOOC. \C~ SC>IOOt. W'IC'.(Olit£A IOIOCX. VCJ'Oil'tM SCl«>OL W:rOltlo" .so.oo&. YJCTOllUI' ICHOOt ~OlltM SCHOOL \'tCT°""'.c:HOQ&. WCTOR!'4iSCHOOl
MCTOftlA SCl«)Ot "9CTOIMSOfOOC. C'r~SOtOO&. WC'T°""""~ VIC~SCHCOL VIC~ to«Xlt YIC'°""' SO<JOt. ~1'04l!IA $O!OOl VIC~.SCHOOt. -.«:-TOl!ltM ~ '1C'"rOlllM.ICHOQl. C-7°""'.tCHOOI..
~r(lllltM .sQ<JiO&. WC:ro.4M SCHOOC. W'letoft".,. lGl-IOClc. \'tero.tM ~ ~"°"""' SQfOOc. l'ICl'OIM KHOO&. \l'JCrOlb& .sGHOOt. we~ SCHOOi. ~'°"""' .S:Cl«Xlt. W:roM&.IO.OC:.. '4C~ ~ v.:r~ ·~
VICTORIA SCHOOL
Write your name, class and register number on all the work you hand in.
Write in dark blue or black pen.
You may use a pencil for any diagrams or graphs.
Do not use paper clips, highlighters, glue or correction fluid .
At the end of the examination , fasten all your work securely together.
The number of marks is given in brackets [ ] at the end of each question or part
question
The total number of marks fo r this paper is 100.
Compound interest
Mensuration
Curved surface a rea of a cone= ,u/
4 '
Volume of a sphere - -1n·
3
Trigonometry
-a- = -b - - -c-
sin A sin B sin C
a2 = b1 + c2 2bccos A
Statistics
Mean = l:fr:
I.f
2 (a) John decides to deposit $180 000 into a bank for 3 years. Orange Bank offers a special
rate of 1.8% per annum simple interest for the first 3 years. Blue Bank also has a special
package which offers interest compounded half yearly at 1.8% per annum. Calculate the
total interest John would receive after 3 years if he deposits all his money into
(b) During the Great Singapore Sale, a table is sold at a 40% discount. A further
reduction of 20% 1s given on the discounted price if a discount coupon is used. A
customer buys a table using a discount coupon. Calculate the price she has to pay
if the original price of the table is $360. [2]
(c) The original price of a car is $70 500. A car can be bought under either one of the
following schemes:
Calculate the difference in the amount paid under the two schemes. (3]
4
3
T s R
In the diagram, PQT and RST are tangents to a circle, centre 0. Angle QTS = 66°,
angle SAB = 33° and angle ACQ = 83°.
Find
Math marks (x) 50 $ x < 60 60 Sx <70 70s x <80 80Sx <90 9os x < ioo I
Number of
students
28 14 18 28 12
I
(a) Calculate the
(b) Two students are chosen at random. Find the probability that one of them
obtained less than 70 marks and the other more than or equal to 90 marks. f2)
(a) Write down an expression, in terms of x, the amount each member of the group
should pay. [1]
One week before the start of the course, 6 students withdrew, but 3 new members signed
up for it. The total cost now increased by $30.
(b) Write down an expression, in terms of x, the amount each member of the group
should now pay. [1]
(c) If the cost per student was $37 more than what was originally expected, form an
equation in x and show that it reduces to 37 x 2 - 14 lx - 6210 =0 . [3]
(e) Hence, find the initial amount each member of the group should pay. [l]
5 6 15
Time (t seconds)
The diagram shows the speed-time graph of a motorcycle and a sports car travelling on the
same stretch of road. Both vehicles started from the same point. The sports car started its
journey from rest. 5 seconds after the motorcycle left and travelled with the same acceleration
as the motorcycle· s initial journey of 6 seconds. The sports car reached the speed of v mis
when t = 15. Find the
(d) time taken for the sports car, after it started its journey, to overtake the
motorcycle. [4]
(b) A signboard, S, is equidistant from JK and JM, and is such that JS= MS. Label
the point Sand measure the distance of the signboard from M. [ 4]
8
A
B p ()
b
In the diagram. OA = a and OB= h. The points Rand P lie on A8 and OB respectively such
- 2- - -
that OP= - PB and AR= kAB. The lines AP and OR intersect at the point Q.
3
(i) AB, [I J
area of L1ABP
(i) ll J
area of t.1AOB
area of tJPRO
(ii) 12]
area of. IA OB
The diagram shows 4 points A, B. C and D on level ground. A is due north of B and
ADC is a straight line. Angle BAD= 38°, angle ADB = 56°, BD = 6 km and CD= 10 km.
(a) Calculate
(b) A vertical tower stands at D. The angle of elevation of the top of the tower, T, from
Bis 2°. Find
(ii) the maximum angle of elevation of the top of the tower, T, when observed
along BC. [2]
,.t,
,. ,. I' I
I
'
''
,. ,. ,. I
I ''
40 ' 60
Ix
Diagram I Diagram II
(a) Diagram I shows a frustum of height x cm formed when a smaller right circular cone
is cut off from a larger one. The height and the base radius of the larger cone are
40 cm and 35 cm respectively.
(i) Show that the radius of the smaller cone cut off is equal to 2(40- x) cm. [2]
8
(i) Calculate, to the nearest cm3 , the amount of water needed to fill the container to
its rim. [4]
(ii) A small circular hole of radius 0.4 cm is drilled through the bottom of the
hemisphere and a cylindrical pipe is attached to it. The water in the container
is drained through this pipe at a constant rate of 1 mis.
Calculate, to the nearest minute, the time taken for the water in the container to
be completely drained off. [3]
The variables x and y are connected by the equation y = x + _?_ _ 4. Some corresponding
x
values of x and y, correct to 2 decimal places, are given in the table below.
x 0.5 1 2 2.5 4 5 6 7
(b) Using a scale of 2 cm to represent 1 unit on both axes, draw the graph of
5
y = x + - - 4 for 0 : : :; x : : :; 7 . [3]
x
(e) By drawing a tangent, find the coordinates of the point on the curve such that
the gradient of the tangent at the point is -1. [2]
End of Paper
This document is intended for internal circulatwn m Vic:trmu School only t>o par/ of this document may be reproduced. stored ma relrleval
system or transmllled m any form or by any mectn~'. el1:ctro111c, mechamcal pho1ocopymg or 01hen1 ise. w11ho111 the prior permtss1011 of the
Victoria School Internal t:xams Comm111ee.
b) x = 2.88 or -5.38 b)
c) (;b:ba) c)
3a) LSQB=33°
b) LBSR =33° 9ai) BC= 14.3 km
c) LTOQ = 57° ii) LDBC =35.6°
d) LABQ = 50° iii) Bearing of B from C = 058.4°
e) LCSO = 17° iv) Shortest distance= 3..t9 km
bi) Height of tower = 210 m
4ai) Mean= 73.2 marks ii) 3.40
ii) SD= 14. l marks
b) Probabilitv = ~
, 275
r1 ( 40-x )3
.i9n 1000--
IOaii) - J cm,
3 6-l
L
bi) 138404 cm 3
$ 2070 ii) 46 minutes
Sa)
x
$ 2100
b)
x-3 lla) a= 0.5,b = 1.25
7 c) Least value of y = 0.4 5
d) x =15.-11-
37 di) x = 0.8 or x = 6.2
e) $138 ii) x $ 0.7 or x ~ 3.9
e) Coordinates (1.5,0.8)
b) s= 15 -l ms- I
9
c) v=20
d) 16.1 seconds
7a)
,VJ L
J K
LJ}v/L = 1-l0°
b) \fS=53 m
8ai) AB = b-a
Mathematical Formulae
Compound interest
Mensuration
Curved surface area of a cone = ;rrl
Trigonometry
a b c
--=--=--
sin A sin B sin C
a2 = b2 + c2 - 2bccosA
Statistics
Mean= l./x
l,f
1 (a) A farmer has l 500 tomato plants. He estimates that each plant will produce 6.5 kg
of tomatoes. Calculate the total mass of tomatoes, in tonnes, that he can expect to
be produced. (1 tonne= I 000 kg] [2]
(b) He proposes to apply 220 ml of liquid fertilizer which is imported from America to
each plant.
The fertilizer is sold in containers each holding 50 litres and costing US$165.
Calculate
(i) the number of containers he must buy, [2]
(ii) the total cost of the fertilizer in Singapore dollars at the exchange rate of
US$I=S$1.43. [2]
(c) The farmer has 0.3 hectares ofland available for the l 500 plants.
Calculate the average area, in square metres, available for each plant.
(1 hectare= 10 000 m2] [2]
(b) Express ED in terms of k and p. Hence, show that ED is parallel to AB. [2]
(d) If the area of the triangle FEB is 10 square units, find the area of the quadrilateral
ABDE. [3]
25:
3
3 (a)
PQRS is a cyclic quadrilateral. PR and QS meet at X RST touches the circle XPS
at S. If angle RQS =65° and angle PST= 110°, find, by stating the reasons
clearly,
(i) angle RPS, [1]
(ii) angle RSX, [l]
(iii) angle PQS. [l]
(b)
•I
•
22: I
I
33
•
[Turn Over
4
2a
(ii) - -x-a
-- - - -2- [3]
(x - 2a) 2 x2- 5ax+6a2 3a - x
I
I
I
A B•
-------------~·--
8
20
75 Time in seconds
0 10 35 55
The diagram shows the speed - time graph of a car over a period of 75 seconds. The car
decelerated uniformly from u rn/s to 20 mis during the first l 0 seconds and continued to
move at constant speed of 20 rn/s for 25 seconds. It accelerated uniformly to a speed of
40 mis in 20 seconds and then decelerated uniformly at a different rate for a further 20
seconds.
Calculate
(a) the speed of the car after 50 seconds, [2]
(b) the deceleration during the last 20 seconds of its motion, [ 1]
(c) the value of u, if the distance travelled in the first 10 seconds is 275 metres, [2]
(d) the distance travelled by the car in the first 55 seconds of its motion. [2]
v v
Figure l
Figure2
A conical funnel of depth 13 cm and vertical angle 84°, is held upright with its vertex V
downwards, as shown in Figure 1.
A freshly painted wooden ball is carefully placed in the funnel. It is observed that the
highest point of the ball, centre 0 and radius r cm, is at the same level as the rim of the
funnel, as shown by the vertical cross-section view in Figure 2. The ball is now
removed and a circular line of paint is found adhering to the funnel.
(b) Show that the length OV is approximately 1.494r cm. [2]
(c) Hence, calculate the radius, r, of the wooden ball. [2]
(d) Find the length of the circular line of paint. [4]
[Turn Over
255
6
8 The table below shows the results of compressive strength tests on a batch of 100
concrete cubes.
(b) Calculate
(i) the mean compressive strength, [2]
(ii) the standard deviation of the compressive strength. [2]
2
(c) If the mean compressive strength of another batch of 100 concrete cubes is 240 kg/cm
and its standard deviation is 18.5 kg/cm 2, compare and comment on these two
batches of concrete cubes. [2]
9
..
North
I
R
North
+
I
I
8
: 35°
p
A ship sails 8 km from P to Qon a bearing of 035°. It then sails 5 km from Q to R on a
bearing of 075°.
(a) Calculate
(i) the angle PQR, (1]
(ii) the distance PR, [3]
(iii) the bearing of R from P. (3)
(b) The ship finally sails from R to a position T, which is due North of Q. Given that
11 A business makes toy buses and toy lorries. The following table is used to calculate the
selling price of each toy.
Labour costs $8 per hour, wood costs $1 per block and paint costs $2 per tin. The
percentage profit made for a bus is 5% of its cost price and for a lorry is 10% of its cost price.
12 (a)
x 480
X t I I I I I
-·-..a--~---·
I I I ·--- ... --'--•-
' I I
- IL - JI - -'-
I - - - . • - - - _1 _ - t.. - .i -
I I I
-T--.--r---·
I
I
I
I
I
I
----.,--r-r- I I I
' I I
80
Diagram I
Square tiles, of side~ x cm, are to be stuck to a wall so that they fill a rectangular
space of 480 cm by 80 cm. Some of the tiles are shown in Diagram t.
(i) Write down an expression, in terms of x, for the number of tiles that will fit
across the top row. [I]
(ii) Given that 600 tiles are required to fill the whole space, find the value of x. [2]
(b)
480
y-5
Diagram 2
Answers:
The End]
9
l(a) 9.75 tonnes (b )(i) 7 containers (ii) $1651.65 (c) 2 m2
--+
2(a) r-q =(k-l)p (b) ED= (2k-l)p (c) 2.!. (d) I 00 unit2
2
3(a)(i) 65° (ii) 65° (iii) 45°
(b)(i) 8cm (ii) $155.56 (iii) 1621
8
S I
x 2
4(a)(i) a2b2 (ii) 2
(b) 3 or -2-
(x-2a) 7
ll(a)
G~) (b) C = (l.05
0
0) ( 60.9)
1.1 ' 35.2
(c) (0.95 0.95); (91.295)
480
12(a)(i) (ii) 8
x
480 5
(b)(i) (iii) 20 or - ; 20 cm by 15 cm
y-5 9
ANDERSON SECONDARY SCHOOL
2011 Preliminary Examination
Secondary Four Express I Four Normal I Five Normal
CANDIDATE
NAME
CENTRE INDEX
NUMBER IsI I I I I NUMBER II I I I
MA TH EMA TICS 4016/02
Paper 2 13 September 2011
2 hours 30 minutes
Additional Materials: Writing paper (10 sheets)
Graph paper (1 sheet)
Write your name, centre number and index number on all the work you hand in.
Write in dark blue or black pen both sides of the paper.
You may use a pencil for any diagrams or graphs.
Do not use staples, paper clips, highlighters, glue or correction fluid.
At the end of the examination, fasten all your work securely together.
The number of marks is given in brackets [ ) at the end of each question or part
question.
The total of the marks for this paper is 100.
Compound Interest
Mensuration
1
Volume of a cone= -nr 2h
3
4
Volume of a sphere= -nr 3
3
Trigonometry
a b c
--=--=
sinA sinB sinC
a2 =b 2 + c 2 - 2bc cos A
Statistics
Mean = 'Lj'(
'Lf
1 The original price of a bag was US$358. After a discount, the sale price of the bag
became US$149.
On a particular day, there was a factory promotion that gave a further 30% discount
on the sale price for the bag.
A tax of 8.75% was imposed on the new sale price of the bag.
6m - 2mn + n 2 - 3n
2 {a) Simplify 2 2
[4]
8m -2mn-n
l lx-3 2x2 -8x
(b) Solve the equation - =0. [4]
x2 +x-12 16-x2
3 The diagram shows a paper weight in the form of a prism where ABCD, ABFE and
CDEF are rectangular faces and ADE and BCF are triangular faces. AB -= 6.5 cm,
A
6.5 cm
F
,,
,
____ ,,,'
B ,' C
(12 - 3x) cm
(a) Given that the difference in the area between the rectangle ABCD and the
triangle ADE is 21.6 cm2 .
Form an equation in x and show that it reduces to l 5x 2 - 23 lx + 468 =0. (2]
(b) Solve the equation 15x 1 - 23 lx + 468 = 0. (2]
(c) Calculate the volume of the paper weight. [2]
4 The diagrams below show the fust three triangles in a sequence of triangles of
increasing size.
Each triangle is formed by joining the neatly arranged dots such that the smaller
right-angled triangles are exactly identical.
The table below shows the number of dots and the number of small right-angled
triangles in each of the larger triangles.
n y z
(a) Write down two matrices such that the elements of their product, under matrix
multiplication, will give the average total amount of ticket sales in each theme
park. Evaluate the product of these l\VO matrices and label the matrix T. [2]
I
(b) Evaluate - (1 I l)T. [1]
3
7 Jn the diagram, LABC = 90 • , AB= 16 cm, BC = 12 cm, BP= x cm and the point Q
on the line AC is the image point of B under tl1e reflection in the line AP.
Calculate, without the use of a calculator, the exact value of
(a) x. [5]
A
16
\' p c
12
8 The distribution below shows the number of SMS messages 20 students sent in a
particular month.
78 74 84 78 85 94 88 72 68 76
25 84 78 82 78 87 80 92 95 69
(c)
•
'
I
-----+------+-----+-----t-------+------
I
I
Number of SMS
mcssagcs
25 a 95
Q p
(a) (i) ShO\.\ that triangle QRS and triangle PST are congruent. [3]
10
- - - 4 l
In the diagram above, OA =a, OB= b and CA =- a - - b.
3 3
Mis the midpoint of AB and AC cuts OB at D.
11
A D
B c
The diagram above shows a playground in the shape of a regular hexagon ABCDEF
with centre 0 A ci rcular fountain with centre X lies in the hexagon such that ll
(a) Calculate
(i) LABC, [I]
(ii) the area of triangle AOB in kilometers square. leaving your ans\ver
in standard form. l2J
(iii) the radius of the fountain. [3]
(b) A boy walks directly from A to E. Calculate the distance he walks. (2]
(i) Calculate the angle of elevation of the bird on top of the lamp post
from A. [ l]
(ii) The boy decides to walk directly back along EA until he reaches a
point T from which the angle of depression of the boy from the bird
is the greatest. Calculate the distance OT. [2 j
22
Container A
(a) Calculate
(i) the volume of Container A, leaving your answer in terms of 7t. [2]
(ii) the surface area of Container A, giving your answer to 2 decimal place. [2]
Container B
A closed hollow Container B which has the same volume as Container A, is made up
of a right pyramid of height 42 cm and a cuboid of height 2 cm. Both the pyramid and
the cuboid have the same square base.
(b) Show that the length of the square base, correct to l decimal place, is 19 .5 cm. [2]
(c) If Container B is filled with water up to ~of its volume, find the depth of water
4
in the container. [4]
Container B
The table below gives the x- and y-coordinates of some points which lie on a curve
y = -x3 + x 2 + 5x + 3.
-3 -1 0 1 2 3 4
I: I 24 0 3 8 9 0 -25
given in the table and join them with a smooth curve. [2]
(c) By drawing a suitable tangent, find the gradient of the curve at the point where
x = l. (2]
(d) The x-coordinates of the points where the line y = -3x + 3 intersects the curve
(e) Using the same axes, draw the line y =-3x + 3. From your graphs, determine
the range of values of x for - 3 ~ x ~ 4 for which - x 3 + x 2 + 5x + 3 > -3x + 3.
[2)
(f) The line y =-~ x + k intersects the curve y =-x3 + x2 + 5x + 3 at least two
2
points. Find the largest value of k. [l]
ANSWER SCHEME
lO(c)(i)
oc 1
-=- 13(b) x=3.3
AB 3
area of l:!.OCD 1
1O(c)(ii) =- J3(c) 4
area of MED 9
1
lO(d) -(4a-b) 13(d) a= 8, b=O
12
area of l:!.OCD l
lO(e)(i) =- 13(e) -3~x<-2.4 0<x<3.3
area of 60CA 4 '
3
lO(e)(ii) - l 3(t) k= 14
2
area of ll.BCD 3
10(e)(iii) =-
area of l:!.OABC 16
ll(a)(i) 120°
l l(a)(ii) l .08x io-3 km2
1 l(a)(iii) 10.4 m
1 l(b) 86.6m
ll(c)(i) 25m
1 l(c)(ii) 10.5°
:
j ......... ........ 2.9
)'
...............................................i ............T..........................,...........................,.............
------·-·-.:"""''
:. . . ....:. ...........~............ l.............;.............
-J -2. s -i r
Oo00••••••• • :• •-•0 •••••• • ~ •••••• ••• • - : ·••OOo.••°'• • :- --• 00 •• 00 •!- 0 000000- o oo~oo ooU oo oUoo 0 • 00 O• U 0_.,0 - : • O>T0 . . •0• ~.·
. 0• 0 0 0••0 O OOroOO O OOOOO oOO~ooOooooooooO•o
~ . ; : ~ ~
··• ···• !· • ·,:··- ··•·• i: .. . ... i. • • -;.I ~ . .. : .; .
~ : ~
. ...... .T...
.
,:.............ti . .
1" . r... . - . .
-~,. . . ····· H •• • • • • •••••
~ l i ' . ;
............ ·1· ............i............. ,.............;--.......... ·1·............:. .. ....... ... ........... "! .............,........... . ~ .............,.... ...... .. ..........·\·...........:·